Paramedic National Test Prep: Medical

¡Supera tus tareas y exámenes ahora con Quizwiz!

A 19-year-old female ingested an unknown quantity of Elavil and Ativan approximately 3 hours ago. She is slow to answer your questions and her speech is slurred. Her BP is 80/60 mm Hg, her pulse is 140 beats/min, and her respirations are 22 breaths/min. The ECG reveals sinus tachycardia with QRS complexes that measure 0.08 seconds. Treatment should include: A: 20 mL/kg sequential IV fluid boluses. B: 0.2 to 0.3 mg of flumazenil IV push. C: 1 mEq/kg of sodium bicarbonate. D: 1 to 2 g/kg of activated charcoal.

*A: 20 mL/kg sequential IV fluid boluses* Reason: Amitriptyline (Elavil) is a tricyclic antidepressant (TCA). In prescribed doses, TCAs are effective in treating depression; when taken in excess, they can be lethal. Common symptoms of TCA toxicity include altered mental status, dysrhythmias (usually sinus tachycardia or supraventricular tachycardia), blurred vision, and pupillary dilation (mydriasis). Severe toxicity causes respiratory depression, QRS and QT interval widening, ventricular dysrhythmias, hypotension, pulmonary edema, and seizures. Treatment includes high-flow oxygen (assist ventilations if the patient's breathing is inadequate), sequential 20 mL/kg IV fluid boluses to treat hypotension (monitor for signs of pulmonary edema), continuous cardiac monitoring, and prompt transport. Sodium bicarbonate is indicated if QRS widening occurs; contact medical control or follow your local protocols regarding the dose. Alkalinizing the blood increases protein binding and may decrease the QRS width, stabilize dysrhythmias, and increase blood pressure. Flumazenil (Romazicon), a benzodiazepine antagonist, should NOT be given to patients who have coingested TCAs and benzodiazepines (ie, lorazepam [Ativan], diazepam [Valium]); it may unmask seizures. Activated charcoal is not indicated for this patient; her symptoms clearly indicate systemic absorption of the drug and her mental status is altered.

You arrive at a local community center where a 30-year-old man has been having a seizure for the past 20 minutes. After properly managing his airway and establishing vascular access, you should give: A: 5 to 10 mg of diazepam. B: 25 g of glucose. C: 100 mg of thiamine. D: 0.4 to 2 mg of naloxone.

*A: 5 to 10 mg of diazepam* Reason: A patient that has been having a seizure continuously for more than 10 minutes is said to be in status epilepticus. After ensuring airway patency and adequate ventilation, your next priority should be to pharmacologically terminate the seizure. Benzodiazepines, such as diazepam (Valium) and lorazepam (Ativan) are effective in terminating seizures. The correct dose of diazepam for seizure termination is 5 to 10 mg. If vascular access cannot be obtained, administer the benzodiazepine via the intramuscular route. If you suspect a narcotic overdose as the underlying cause of the seizure, given naloxone (Narcan) accordingly. If hypoglycemia is documented, administer 25 g of 50% dextrose. Thiamine (vitamin B1) is not routinely given to patients who are experiencing a seizure, although it should be considered if your patient has a history of alcoholism and is hypoglycemic.

Which of the following assessment findings is MOST indicative of peritoneal irritation? A: A decrease in pain when drawing the knees into the abdomen B: Pain that is referred to the shoulder, neck, or scapulae C: Pain that increases when the patient is placed on his or her side D: A relief of pain when the patient moves around frequently

*A: A decrease in pain when drawing the knees into the abdomen*

Which of the following patients is at greatest risk for suicide? A: A woman whose depression acutely improves B: A man who owns multiple guns and knives C: A woman whose mother committed suicide D: A man who has not slept for over 72 hours

*A: A woman whose depression acutely improves* Reason: Risk factors for suicide include depression, major negative life changes (ie, financial hardship, loss of a loved one, loss of a job), and marital problems, among others. You should be especially alert for the patient whose depression, especially chronic depression, acutely improves. This is a red flag indicator that the patient has developed a plan that he or she is comfortable with, and that he or she feels as though resolution is near.

A 23-year-old male is found unresponsive. According to a friend, the patient had a headache and said that he was going to take a nap. His breathing is rapid and shallow, his pulse is rapid and weak, and he is profusely diaphoretic. Which of the following represents the MOST appropriate treatment for this patient? A: Assist ventilations with a bag-mask device, assess his blood glucose level, apply the cardiac monitor, obtain vital signs, and establish vascular access. B: Apply oxygen via nonrebreathing mask, start an IV line, administer 25 g of 50% dextrose, apply a cardiac monitor, and obtain vital signs. C: Preoxygenate him and prepare to intubate, start an IV line, administer 2 mg of naloxone, assess his vital signs, and apply the cardiac monitor. D: Hyperventilate him with a bag-mask device, conduct an in-depth neurologic exam, obtain a 12-lead ECG, and administer 1 mg of glucagon IM.

*A: Assist ventilations with a bag-mask device, assess his blood glucose level, apply the cardiac monitor, obtain vital signs, and establish vascular access* Reason: The patient's respirations are likely not producing adequate minute volume and should be assisted with a bag-mask device. Hyperventilation should be avoided; it increases the incidence of gastric distention and may impair venous return to the heart due to increased intrathoracic pressure. You must next try to determine the cause of his unresponsiveness. Tachypnea and tachycardia are not consistent with a narcotic overdose; therefore, naloxone (Narcan) will likely have no effect. He may be hypoglycemic; however, before administering dextrose or glucagon, assess his blood glucose level first. Cardiac monitoring is indicated in order to detect life-threatening dysrhythmias; obtain a 12-lead ECG when possible. Vascular access is indicated in case fluid boluses or drug therapy is necessary. Intubation may be necessary if the patient requires prolonged ventilatory support or if you are unable to effectively ventilate with a bag-mask device. The ability to rule out (or in) causes of altered mental status in the field is relatively limited; therefore, rapid transport is essential.

What is the pathophysiologic difference between cyanide and carbon monoxide? A: Carbon monoxide binds with the hemoglobin molecule. B: Cyanide destroys the cells of the immune system. C: Carbon monoxide destroys the red blood cells. D: Cyanide attaches to the hemoglobin molecule.

*A: Carbon monoxide binds with the hemoglobin molecule* Reason: Carbon monoxide (CO) binds to the hemoglobin molecule and inhibits the oxygen carrying ability of the blood. CO has an affinity for hemoglobin that is 200 to 250 times greater than that of oxygen. Cyanide blocks the uptake of oxygen at the cellular level. Both result in inadequate oxygenation and cellular death if left untreated.

Which of the following conditions would produce the MOST rapid loss of consciousness? A: Insulin shock B: Hyperglycemia C: Ketoacidosis D: Ischemic stroke

*A: Insulin shock* Reason: Insulin shock (hypoglycemic crisis) is most noted for its rapid onset of symptoms, which includes loss of consciousness. Hyperglycemia, which can lead to ketoacidosis, typically presents over a period of hours to days. Unlike a hemorrhagic stroke, an ischemic stroke generally does cause a rapid or immediate loss of consciousness.

Which of the following is a defining factor in the transition from human immunodeficiency virus (HIV) infection to acquired immunodeficiency syndrome (AIDS)? A: Development of opportunistic infections B: Antibodies are detected in the blood C: Transient increase in the T-cell count D: Fever that lasts greater than 7 days

*A: Development of opportunistic infections* Reason: Acquired immunodeficiency syndrome (AIDS) is the end-stage disease process caused by infection with the human immunodeficiency virus (HIV). Although it can obviously be stated that all patients with AIDS are HIV positive, it cannot be stated that all HIV-positive patients have AIDS. A diagnosis of AIDS is made when the number of T-helper lymphocytes (CD4 cells) fall below a certain level, or when the patient develops one of a specific group of opportunistic infections (also called AIDS-defining or AIDS-related conditions). Such opportunistic infections include Pneumocystis carinii pneumonia, cytomegalovirus, red or purple malignant skin lesions called Kaposi sarcoma, atypical tuberculosis, and cryptococcol meningitis. The point at which antibodies to the virus are detected in the blood is called seroconversion; this usually occurs within the first 3 months following exposure to HIV. With antiretroviral therapy, patients can remain HIV-positive for many years without showing any evidence of infection.

Which of the following is a defining factor in the transition from human immunodeficiency virus (HIV) infection to acquired immunodeficiency syndrome (AIDS)? A: Development of opportunistic infections B: Fever that lasts greater than 7 days C: Antibodies are detected in the blood D: Transient increase in the T-cell count

*A: Development of opportunistic infections* Reason: Acquired immunodeficiency syndrome (AIDS) is the end-stage disease process caused by infection with the human immunodeficiency virus (HIV). Although it can obviously be stated that all patients with AIDS are HIV positive, it cannot be stated that all HIV-positive patients have AIDS. A diagnosis of AIDS is made when the number of T-helper lymphocytes (CD4 cells) fall below a certain level, or when the patient develops one of a specific group of opportunistic infections (also called AIDS-defining or AIDS-related conditions). Such opportunistic infections include Pneumocystis carinii pneumonia, cytomegalovirus, red or purple malignant skin lesions called Kaposi sarcoma, atypical tuberculosis, and cryptococcol meningitis. The point at which antibodies to the virus are detected in the blood is called seroconversion; this usually occurs within the first 3 months following exposure to HIV. With antiretroviral therapy, patients can remain HIV-positive for many years without showing any evidence of infection.

A 49-year-old female complains of severe neck pain. Her head is turned to the side and appears locked in that position. You also note that her eyes are clenched shut. She is conscious, but agitated. Her husband tells you that she started taking lithium the day before after being diagnosed with bipolar disorder. Based on this patient's clinical presentation, which of the following medications would MOST likely be indicated? A: Diphenhydramine B: Haloperidol C: Naloxone D: Calcium chloride

*A: Diphenhydramine* Reason: The patient is experiencing drug-induced dystonia. Dystonia is marked by severe muscle spasms that cause bizarre contortions, repetitive motions, or postures (localized or diffuse). Patients who take antipsychotic or neuroleptic drugs may experience an acute onset of bizarre contortions of the face or body (secondary dystonia), usually within 12 to 36 hours after a new drug is started or the dose of an existing drug is increased. Drugs commonly associated with dystonia include lithium, phenytoin (Dilantin), antidepressants, reserpine (Serpalan), and carbamazepine (Tegretol), among others. Numerous patterns of drug-induced dystonia exist; this patient has two of the most common: blepharospasm (eyelid muscle spasm that causes forced eyelid closure) and torticollis (neck muscle spasms that force the head to the side). Drug-induced dystonic reactions are treated with diphenhydramine (Benadryl), 50 mg IV over 2 minutes (repeat in 15 to 30 minutes if needed). Dopamine blockade by antipsychotics disrupts the ratio of acetylcholine and dopamine in the brain; anticholinergic drugs (eg, diphenhydramine) may restore this ratio. Haloperidol (Haldol), an antipsychotic drug, may cause dystonia and is clearly contraindicated. Naloxone (Narcan) reverses the CNS depressant effects of narcotic (opiate) drugs, and calcium chloride is used to treat hyperkalemia and calcium-channel blocker overdose; neither is used to treat dystonia.

Which of the following is MOST indicative of hyperglycemic ketoacidosis? A: Hyperpnea B: Acute onset C: Diaphoresis D: Bradypnea

*A: Hyperpnea* Reason: When glucose does not reach the cell where it can be used for energy, the cell will metabolize fat instead, which produces ketoacids. As the body attempts to rid itself of these acids, the respirations become deep (hyperpnea) and rapid (tachypnea) with a noted fruity or acetone breath odor (Kussmaul respirations). Hyperglycemic ketoacidosis manifests over several hours to even a few days. As blood glucose levels rise, the kidneys attempt to excrete the excess glucose, taking water with it. This results in dehydration, which typically manifests with warm, dry skin that has poor turgor.

Which of the following statements regarding sickle cell disease is correct? A: In sickle cell disease, misshapen red blood cells can lodge in the spleen, causing it to swell and rupture. B: The red blood cells in sickle cell disease have a rounded appearance, rather than the normal oblong appearance. C: When the sickle cell gene is inherited from both parents, the patient will probably not develop the disease. D: Despite their altered shape, the red blood cells of a person with sickle cell disease are effective carriers of oxygen.

*A: In sickle cell disease, misshapen red blood cells can lodge in the spleen, causing it to swell and rupture* Reason: Sickle cell disease is the most commonly inherited blood disorder. Although it primarily affects African-American, Puerto Rican, and European populations, it can occur in anyone. Sickle cell disease starts with a gene defect of the adult-type hemoglobin (HbA). This mutated gene can be inherited from one parent (HbS) or both parents (HbSS). When the gene is inherited from both parents, there is a high probability that the patient will be prone to sickle cells (that is, the person actually has the disease) or the sickle cell trait (the person is a carrier of the mutated gene). The defective red blood cells (RBCs) are misshapen; affected cells have an oblong (sickle) shape instead of a smooth, round shape. This shape makes the RBCs poor carriers of oxygen, which means that a person with sickle cell disease is highly susceptible to hypoxia. The odd shape may also cause RBCs to lodge in small blood vessels (thrombotic crisis) or in the spleen, causing the organ to swell and rupture, which can cause death. Sickle cell disease may also lead to other crises, such as aplastic crisis and hemolytic crisis. In aplastic crisis, RBC production temporarily stops. In hemolytic crisis, the RBCs break down quickly. In acute crises, patients often have severe pain, which will require aggressive pain management. They may also experience frequent infections, which can lead to sepsis and death.

What is a common finding in both fresh water and salt water drownings? A: Inadequate oxygenation B: Loss of surfactant C: Severe metabolic alkalosis D: Pulmonary edema

*A: Inadequate oxygenation* Reason: Though the mechanisms are different in salt water drownings as opposed to fresh water drownings, inadequate oxygenation, which leads to hypoxia and metabolic acidosis, is common to both and is typically the result of laryngospasm.

A young male intentionally ingested unknown quantities of ibuprofen, aspirin, Lortab, and Anafranil. He is unresponsive, and is hypoventilating, bradycardic, and hypotensive. Which of these drugs is the MOST likely cause of his clinical presentation? A: Lortab B: Ibuprofen C: Anafranil D: Aspirin

*A: Lortab* Reason: Of the drugs listed, Lortab (acetaminophen [APAP] and hydrocodone) is the most likely cause of this patient's clinical presentation. Hydrocodone is a narcotic analgesic, and when taken in excess, it can result in central nervous system depression (ie, altered mental status, hypoventilation, bradycardia, hypotension). Aspirin toxicity causes metabolic acidosis and would likely manifest with hyperventilation, an indication that the respiratory buffer system is attempting to eliminate acids from the body. Clomipramine (Anafranil) is a tricyclic antidepressant and would likely result in tachycardia when taken in excess. Ibuprofen, the active ingredient in Advil and Motrin, errodes the gastric lining and can be toxic to the liver; CNS depression is not indicative of ibuprofen toxicity.

Which of the following is MOST indicative of an infectious or communicable disease? A: Persistent fever and night sweats B: Sore throat and nasal discharge C: Vomiting and diarrhea for 3 days D: Severe headache and photophobia

*A: Persistent fever and night sweats* Reason: A persistent fever and night sweats should alert the paramedic to the possibility of a communicable or infectious disease. HIV and tuberculosis both present with these symptoms. Vomiting, diarrhea, sore throat, and a headache could indicate a variety of illnesses, not all of which are communicable or infectious.

A 30-year-old female is found unresponsive by her roommate. According to the roommate, the patient, who has type I diabetes, was fine 30 minutes ago. On assessment, the patient has rapid, shallow respirations; pallor and diaphoresis; and a weak, thready pulse. Additional assessment of this patient will MOST likely reveal: A: a blood glucose level less than 70 mg/dL. B: peaked T waves and wide QRS complexes. C: an acetone or fruity odor on her breath. D: marked hyperglycemia.

*A: a blood glucose level less than 70 mg/dL* Reason: This patient's clinical presentation—pallor, diaphoresis, tachypnea, tachycardia, unresponsiveness—is classic for insulin shock (hypoglycemic crisis). The fact that she was fine 30 minutes ago indicates that her condition occurred acutely, another hallmark feature of insulin shock. When you assess her blood glucose level, it will likely read well below 70 mg/dL (hypoglycemia). Unlike insulin shock, diabetic ketoacidosis (DKA) progresses slowly, over 12 to 48 hours, and is the result of hyperglycemia. In DKA, blood glucose levels of 400 mg/dL or higher are not uncommon. Other signs of DKA include deep, rapid respirations with a fruity or acetone breath odor (Kussmaul respirations), which is a response of the respiratory system to eliminate ketoacids from the blood; warm, dry skin, which is the result of dehydration secondary to the osmotic diuresis caused by hyperglycemia; and a rapid, full pulse, which is also the result of dehydration. Changes in serum potassium are possible in patients with DKA. The associated osmotic diuresis may cause dangerously low potassium levels (hypokalemia), resulting in marked myocardial instability. On the ECG, hypokalemia may manifest with flattened T waves, prominent U waves, ST segment depression, and prolongation of the PR and QT intervals.

A hiker was bitten on the leg by a rattlesnake approximately 20 minutes ago. He complains of pain to the lateral aspect of his left leg, just proximal to the ankle. Assessment of that area reveals two small puncture wounds surrounded by edema. You also note localized twitching of his calf muscles. He is conscious and alert, but anxious. His blood pressure is 112/70 mm Hg, pulse is 120 beats/min, and respirations are 24 breaths/min with adequate depth. In addition to keeping him calm, you should: A: administer high-flow oxygen via nonrebreathing mask, apply a sterile dressing to the wound, immobilize his leg and keep it below the level of the heart, start an IV line and set it at a KVO rate, and transport. B: administer high-flow oxygen via nonrebreathing mask, apply a tight constricting band proximal to the wound, cover the wound with a sterile dressing, start an IV line and set it at a KVO rate, and transport. C: apply a chemical icepack to the wound to decrease venom absorption, immobilize his leg and keep it below the level of the heart, administer high-flow oxygen via nonrebreathing mask, and transport. D: place him in a supine position, assist his ventilations with a bag-mask device, immobilize his leg and elevate it 6" to 12", start an IV line and give a 20 mL/kg crystalloid bolus, and transport.

*A: administer high-flow oxygen via nonrebreathing mask, apply a sterile dressing to the wound, immobilize his leg and keep it below the level of the heart, start an IV line and set it at a KVO rate, and transport* Reason: Pit viper venom contains a mix of enzymes that cause local tissue necrosis, hemolysis, increased vascular permeability, coagulopathy, and neuromuscular dysfunction. Local signs of envenomation include puncture marks, pain, edema, and fasciculations (small, local, involuntary muscle twitching). Systemic signs include nausea and vomiting, dizziness, weakness, numbness or tingling of the mouth and tongue, tachycardia, hypotension, and tachypnea. After ensuring scene safety, ensure that the patient has a patent airway and keep him calm to decrease venom spread and absorption. Your patient, although tachypneic, has adequate tidal volume. Give high-flow oxygen via nonrebreathing mask, but be ready to assist his ventilations. Establish vascular access and set the flow rate to keep the vein open; monitor his BP and give fluid boluses as needed. Cover the wound with a sterile dressing, immobilize his leg, and keep it below the level of the heart. Do NOT apply ice to a snakebite wound; this causes vasoconstriction and may increase venom absorption. Transport to a facility where he can receive antivenin (eg, CroFab).

The clinical presentation of thyroid storm MOST closely resembles that of: A: amphetamine use. B: myxedema. C: ketoacidosis. D: heroin overdose.

*A: amphetamine use* Reason: Thyroid storm is a life-threatening condition that may occur in patients with hyperthyroidism. Hyperthyroidism is a condition in which the thyroid gland produces too much of the thyroid hormones triiodothyronine (T3) and thyroxine (T4). T3 and T4 play a key role in controlling the body's basal metabolic rate (BMR). When produced in excess, these thyroid hormones produce signs and symptoms of adrenergic hyperactivity (ie, tachycardia, sweating, palpitations, anxiety). In thyroid storm, these clinical signs would be more severe, including profound tachycardia, dysrhythmias, agitation and paranoia, and cardiovascular collapse. Amphetamines also cause adrenergic hyperactivity; of the choices listed, amphetamine use and thyroid storm would produce the most similar clinical signs. Heroin, a narcotic, causes central nervous system depression, resulting in a decreased level of consciousness, hypoventilation, bradycardia, and hypotension. Advanced hypothyroidism, called myxedema, results from a severe deficiency of T3 and T4. In contrast to thyroid storm, myxedema does not present with signs of adrenergic hyperactivity.

A man was trapped in his burning house for approximately 15 minutes before firefighters rescued him. He reports burning in his throat and a severe headache. He has a blood pressure of 180/90 mm Hg, a pulse rate of 120 beats/min, and labored respirations of 28 breaths/min. In addition to providing supplemental oxygen, treatment should include: A: amyl nitrite inhaled in 20-second increments. B: IV administration of 1 mL/kg of ethyl alcohol. C: rapid transport to a local hyperbaric facility. D: a slow infusion of sodium nitroprusside.

*A: amyl nitrite inhaled in 20-second increments* Reason: A burning sensation in the mouth or throat, headache, and sympathetic hyperactivity (hypertension and tachycardia) are indicative of cyanide toxicity. The odor of bitter almonds on the patient's breath, while highly suggestive of cyanide toxicity, is not diagnostic. Cyanide acts as a cellular asphyxiant, inhibiting oxygen uptake by the tissues. This causes severe hypoxia, even in the presence of oxygen. After removing the patient from the source, and after ensuring airway patency and breathing adequacy, treatment includes having the patient inhale amyl nitrite for 20 seconds, followed by high-flow oxygen for 40 seconds. Thus, in each minute, the patient should inhale the amyl nitrite one third of the time and breathe oxygen two thirds of the time. Keep switching the amyl nitrite and oxygen while maintaining this ratio. Many of the same burning chemicals that produce carbon monoxide (CO) produce cyanide. Treatment for CO exposure includes high-flow oxygen and possibly hyperbaric therapy, depending on the patient's carboxyhemoglobin (COHb) level. Sodium nitroprusside (Nipride) is a potent vasodilator. Patients receiving long-term Nipride therapy can develop cyanide toxicity, so Nipride is clearly NOT indicated for this patient. Ethyl alcohol (ethanol) may be given at the hospital to treat ethylene glycol (ie, antifreeze) ingestion; it is not given for cyanide toxicity.

Multiple sclerosis is a disease caused by: A: an autoimmune disorder in which the body attacks the myelin sheath of the neurons in the brain and spinal cord. B: inflammation of the trigeminal nerve that leads to deterioration of the myelin sheath and causes severe, chronic pain. C: dysfunction or damage to the portion of the brain that is responsible for the production of dopamine. D: a genetic disorder in which defective DNA causes an error in muscle tissue, such that the malformed muscle cells rupture.

*A: an autoimmune disorder in which the body attacks the myelin sheath of the neurons in the brain and spinal cord* Reason: Multiple sclerosis (MS) is an autoimmune disorder in which the immune system recognizes the protein that makes up the myelin sheath—the protective insulation that coats the axons of most nerve cells (neurons)—as being foreign and creates antibodies, called autoantibodies, that destroy it. This leads to areas of scarring that produce symptoms such as muscle weakness; impairment of pain, temperature, and touch senses; pain (moderate to severe); ataxia; tremors; and speech disturbances. Parkinson's disease results from damage to or dysfunction of the substantia nigra—the portion of the brain that produces dopamine. Patients with Parkinson's disease classically present with fine muscle tremors. A genetic disorder in which defective DNA causes an error in muscle tissue, such that the malformed muscle cells rupture more easily, is called muscular dystrophy (MD). MD typically presents with progressive muscle weakness, delayed development of muscle motor skills, ptosis (drooping of the upper eyelid), drooling, and poor muscle tone. Trigeminal neuralgia—also called tic douloureux—is an inflammation of the trigeminal nerve (fifth cranial nerve). The trigeminal nerve receives sensory information from the face. The usual cause of trigeminal neuralgia is irradiation by an artery lying too close to the nerve. Over time, as the artery changes diameter to meet blood supply needs, the myelin sheath is grated off the nerve. With its insulation gone, the nerve may "short circuit," causing severe shock-like or stabbing pain, usually on one side of the face.

You are called to transport a patient from an urgent care clinic to the emergency department. When reviewing the lab results from the clinic, you note that the patient's thyroid-stimulating hormone (TSH) level is very high. This indicates: A: an underactive thyroid. B: elevated T3/T4 levels. C: Cushing syndrome. D: Graves disease.

*A: an underactive thyroid*

You respond to a residence for an overdose. While en route, the dispatcher advises you that CPR is in progress. A second paramedic unit is dispatched, but their response to the scene will be delayed. When you arrive at the scene, an emergency medical responder is performing CPR on the patient, a 30-year-old male, who apparently ingested an unknown quantity of Darvon. After confirming apnea and pulselessness, you should direct resumption of CPR and then: A: apply the ECG electrodes and assess the patient's cardiac rhythm. B: establish vascular access and administer 2 mg of naloxone. C: locate the medication bottle to confirm what he evidently ingested. D: insert an advanced airway device to ensure adequate ventilation.

*A: apply the ECG electrodes and assess the patient's cardiac rhythm* Reason: After confirming cardiac arrest, resume CPR and assess the patient's cardiac rhythm. Although he ingested propoxyphene (Darvon), a narcotic, your first priority is to determine the need for defibrillation. If indicated, deliver a single shock and immediately resume CPR starting with chest compressions. Reassess the patient every 2 minutes and repeat defibrillation as needed. With your limited resources (delayed response of the backup crew), you must prioritize additional treatment. If bag-mask ventilation (with an oral airway inserted) is adequate, establish vascular access. If bag-mask ventilations are inadequate, insert an advanced airway device and then establish vascular access. After vascular access is established, give a vasopressor (epinephrine or vasopressin). Further treatment should be guided by the patient's cardiac rhythm. Naloxone (Narcan), in a dose of 2 mg, should be given early. When the backup crew arrives, assign someone to search for medication bottles. Whether your resources are few or many, it is critical to perform high-quality CPR with limited interruptions and to defibrillate every 2 minutes if indicated.

You enter an elderly man's residence and find him sitting on the couch with his eyes closed. His respirations appear to be deep and rapid. You should: A: assess his mental status. B: quickly move him to the floor. C: manually open his airway. D: check for a carotid pulse.

*A: assess his mental status* Reason: Just because a patient's eyes are closed does not mean that he or she is unresponsive. The primary assessment of a patient begins by assessing his or her mental status. In this particular case, if you find that the patient is unresponsive, you should quickly move him to the floor, continue with your primary assessment, and begin treatment as needed. If the patient is responsive, continue your primary assessment and then determine his chief complaint.

You are dispatched to a residence for a patient having a seizure. Upon arriving at the scene, you find that the patient, a 39-year-old male, is experiencing a generalized tonic-clonic seizure and is cyanotic. His wife tells you that he has been like this since she called 9-1-1. In addition to protecting the patient from injury, you should: A: assist his breathing with a bag-mask device, establish vascular access, and administer 5 mg of diazepam. B: apply high-flow oxygen via nonrebreathing mask, place him on his side, and wait for the seizure to stop. C: intubate him to prevent aspiration, establish IV or IO access, and administer 0.1 mg/kg of lorazepam. D: administer 0.5 mg/kg of diazepam rectally, cover him with a blanket, and administer high-flow oxygen.

*A: assist his breathing with a bag-mask device, establish vascular access, and administer 5 mg of diazepam* Reason: Status epilepticus is defined as a prolonged (> 10 minutes) seizure or consecutive seizures that occur without an intervening return to consciousness. During a seizure, neurons are using huge amounts of glucose and producing lactic acid. For short periods, this does not cause long-term damage. If the seizure continues, however, the body can't remove the waste products effectively or ensure adequate glucose supplies; this can cause neuronal damage or death. Hypoventilation or apnea is common during a seizure; cyanosis is also common. In short-duration seizures, these are usually short-lived (< 30 seconds) and generally do not require assistance. However, status epilepticus causes prolonged hypoventilation or apnea and requires ventilation assistance, although this may be difficult. Patients experiencing a seizure often have clenched teeth (trismus); therefore, intubation will be nearly impossible and is generally contraindicated. Benzodiazepines (ie, Valium, Ativan) are used to terminate seizures. The dose of Valium is 5 mg IV or IM (repeated every 10 to 15 minutes up to 30 mg). If vascular access is not available, Valium can be given rectally (adult dose, 0.2 mg/kg). The dose of Ativan is 0.05 mg/kg (maximum single dose of 4 mg); this may be repeated in 10 to 15 minutes if needed. If benzodiazepines do not quickly terminate the seizure and the patient cannot be ventilated, rapid-sequence intubation (RSI) may be required.

A 34-year-old woman overdosed on amitriptyline. She is unresponsive and has slow, shallow breathing. Her BP is 70/40 mm Hg and her heart rate is 140 beats/min. The cardiac monitor reveals sinus tachycardia. Initial treatment for her should include: A: assisted ventilation with a bag-mask device and 20 mL/kg normal saline boluses. B: endotracheal intubation and 1 to 2 mEq/kg of sodium bicarbonate IV push. C: high-flow oxygen via nonrebreathing mask and 0.4 to 2 mg of naloxone IV push. D: assisted ventilation with a bag-mask device and 0.2 mg of flumazenil IV push.

*A: assisted ventilation with a bag-mask device and 20 mL/kg normal saline boluses* Reason: Slow, shallow breathing will not produce adequate minute volume and should be treated with assisted ventilation. The patient may require endotracheal intubation, but you must restore minute volume first (think basic!). Amitriptyline is a tricyclic antidepressant (TCA); naloxone (Narcan), a narcotic antagonist, will not reverse its effects. Treat the patient's hypotension with sequential normal saline boluses (20 mL/kg); monitor closely for signs of pulmonary edema. Flumazenil (Romazicon), a benzodiazepine antagonist, should not be given to patients with a TCA overdose; it may cause seizures. Provide continuous ECG monitoring and watch for widening of the QRS complex. If QRS widening occurs, consult with medical control regarding the administration of sodium bicarbonate.

A 19-year-old male experienced a syncopal episode after several hours of vigorous exercise in the heat. Upon arrival at the scene, bystanders direct you to the patient, who they moved under a tree. The patient moans in response to painful stimuli, and his skin is flushed, hot, and moist. After ensuring airway and breathing adequacy, your MOST immediate action should be to: A: begin rapid cooling measures. B: infuse 500 mL of IV fluid. C: assess his blood glucose level. D: take his temperature orally.

*A: begin rapid cooling measures* Reason: This patient is experiencing exertional heat stroke secondary to prolonged exposure to a hot environment. His markedly decreased level of consciousness and hot skin indicate a dangerously high core body temperature (CBT). Because the extent of brain damage depends on the severity and duration of hyperthermia, rapid cooling measures must begin immediately. Otherwise, the patient's CBT will continue to rise and he will die. Remove the patient's clothing (they can trap heat), and begin fanning him while keeping his skin wet. The use of ice packs or ice cold water should be avoided, however, because they may precipitate shivering and cause a further increase in his CBT. Load the patient into the ambulance and begin rapid transport, continuing cooling measures en route. Heat stroke is generally associated with volume depletion; however, IV therapy should be performed en route to the hospital. Although concomitant hypoglycemia is unlikely, you should assess the blood glucose level of any patient with an altered mental status. When assessing the patient's CBT, a hyperthermia thermometer (ie, a rectal probe) should be used, if available. Taking his temperature orally will yield little additional information. Unlike classic heat stroke, which commonly affects young children, elderly patients, and those with significant medical problems, exertional heat stroke does not always present with dry skin. Although the patient is not actively sweating, the skin is often moist from residual perspiration.

In contrast to an anaphylactic reaction, an anaphylactoid reaction: A: can occur without prior exposure to an offending agent. B: usually does not respond to antihistamine medications. C: is an immune response mediated by IgE antibodies. D: is usually less severe and does not require drug therapy.

*A: can occur without prior exposure to an offending agent* Reason: Anaphylaxis is an immune response that is mediated by IgE antibodies, whereas an anaphylactoid reaction is an immune response that does not involve IgE antibody mediation. Anaphylaxis requires the patient to be sensitized first; he or she must be exposed to the offending agent (antigen) in order to build up IgE antibodies. Because an anaphylactoid reaction is not IgE antibody mediated, prior exposure (sensitization) to the offending agent is not required. Examples of common agents that cause an anaphylactoid reaction include some contrast dyes given before radiography, morphine-derivative medications, and aspirin. Even though the process that causes an anaphylactoid reaction is different from an anaphylactic reaction, the clinical presentation, negative consequences (respiratory and circulatory failure), and treatment are the same. Do not discount a severe allergic reaction just because a patient, who presents with suggestive signs and symptoms, tells you that he or she was never exposed to the suspected trigger.

A 39-year-old man is unresponsive, pulseless, and apneic after being lost in the woods during the middle of winter. He has a core body temperature of 85°F (29.4°C). When treating this patient, you should avoid: A: cardiac medications. B: defibrillation. C: passive rewarming. D: intubation.

*A: cardiac medications* Reason: Treatment for patients with cardiac arrest and severe hypothermia (core body temperature less than 86°F [30°C]) includes CPR, intubation (or an alternative airway device), limiting defibrillation to one attempt if the patient is in ventricular fibrillation or pulseless ventricular tachycardia, passive rewarming, and active external rewarming (ie, heating blankets, radiant heat from hot packs). Active internal (core) rewarming is usually performed at the hospital. Cardiac medications should be withheld for two reasons: 1) the patient's metabolic rate is too slow to distribute the drugs, and 2) medications can accumulate to toxic levels in the severely hypothermic patient, which can be detrimental as the patient is rewarmed.

A 39-year-old man is unresponsive, pulseless, and apneic after being lost in the woods during the middle of winter. He has a core body temperature of 85°F (29.4°C). When treating this patient, you should avoid: A: cardiac medications. B: passive rewarming. C: defibrillation. D: intubation.

*A: cardiac medications* Reason: Treatment for patients with cardiac arrest and severe hypothermia (core body temperature less than 86°F [30°C]) includes CPR, intubation (or an alternative airway device), limiting defibrillation to one attempt if the patient is in ventricular fibrillation or pulseless ventricular tachycardia, passive rewarming, and active external rewarming (ie, heating blankets, radiant heat from hot packs). Active internal (core) rewarming is usually performed at the hospital. Cardiac medications should be withheld for two reasons: 1) the patient's metabolic rate is too slow to distribute the drugs, and 2) medications can accumulate to toxic levels in the severely hypothermic patient, which can be detrimental as the patient is rewarmed.

You are dispatched to a residence for a patient with respiratory distress. The patient, a 59-year-old female, has a tracheostomy tube in place and is ventilator dependent. She has marked respiratory distress, perioral cyanosis, and an oxygen saturation of 80%. You should: A: disconnect her from the mechanical ventilator and provide manual positive-pressure ventilation. B: suspect tracheostomy tube dislodgement and quickly remove the tube as the patient exhales. C: detach the ventilator tubing from the tracheostomy and suction the tracheostomy tube for 10 seconds. D: adjust the settings on the mechanical ventilator to ensure that she is receiving adequate tidal volume.

*A: disconnect her from the mechanical ventilator and provide manual positive-pressure ventilation* Reason: When caring for a ventilator-dependent patient whose condition deteriorates, you must rapidly troubleshoot and correct the problem in order to prevent further deterioration. The quickest intervention to perform involves simply disconnecting the mechanical ventilator, attaching a bag device to the tracheostomy tube, and providing manual positive-pressure ventilations. If the patient's condition improves, then equipment malfunction was the likely cause of his or her deterioration. If the patient's condition does not improve despite manual positive-pressure ventilation, the tracheostomy tube may be occluded with thick secretions and will require suctioning. Avoid adjusting home ventilator settings unless you have been specifically trained to work with the particular device (most paramedics are not). Soliciting the help of the patient's caregiver can assist in assessment and troubleshooting of equipment.

A known heroin abuser is found unresponsive by a law enforcement officer. Your primary assessment of the patient, a 24-year-old female, reveals that she is unresponsive, is breathing at a rate of 6 breaths/min and shallow, and has a pulse rate of 40 beats/min and weak. You should: A: ensure that her airway is clear and begin assisting her ventilations. B: begin immediate cardiac pacing to increase her heart rate. C: administer high-flow oxygen, start an IV, and give her atropine. D: intubate her trachea, start an IV line, and give her naloxone.

*A: ensure that her airway is clear and begin assisting her ventilations* Reason: The patient's airway must be clear and adequate oxygenation and ventilation must be established; otherwise, she will die. Respirations of 6 breaths/min and shallow (reduced tidal volume) will not provide adequate minute volume and require immediate treatment. You must first ensure that her airway is clear of secretions and assist her ventilations with a bag-mask device to increase rate, tidal volume, and minute volume. After adequate oxygenation and ventilation have been established, apply the cardiac monitor and establish vascular access. Given her history and clinical presentation—gross CNS depression—naloxone (Narcan) would be the most appropriate initial drug to administer; however, because she is a known heroin abuser, consider giving 0.4 mg instead of the standard 2 mg dose; this will minimize the risk of inducing an acute withdrawal seizure. If she remains bradycardic despite naloxone administration, prepare for immediate transcutaneous cardiac pacing (TCP); atropine (0.5 mg) should also be considered. In patients with hypoventilation secondary to a drug overdose, support ventilations with a bag-mask device first. If the patient's condition is refractory to naloxone, intubation should then be considered, particularly if your transport time will be prolonged. In many cases, bag-mask ventilations and naloxone will improve the patient's ventilatory status, thus avoiding the need for intubation. Remember that most narcotics outlive a single dose of naloxone and CNS depression may recur; therefore, repeat dosing with naloxone is often necessary.

You are called to a local park where a 30-year-old man has collapsed. When you arrive, you find that the patient is disoriented. His skin is hot and moist and his respirations are rapid and shallow. You should suspect: A: heatstroke. B: heat cramps. C: heat exhaustion. D: heat prostration.

*A: heatstroke* Reason: Hot (dry or moist) skin, tachypnea, and an altered mental status are indicative of heatstroke. Two major factors help differentiate heat exhaustion from heatstroke. In heat exhaustion, sometimes called heat prostration, the patient is usually conscious and alert with cool, moist skin. In heatstroke, the patient's mental status is typically altered and his or her skin is hot to the touch. Although the patient in heatstroke is not actively sweating, he or she may have residual perspiration on the skin; this is especially true in patients with extertional heatstroke. Do not rule out heatstroke just because the patient's skin is moist.

Pickwickian syndrome is a condition in which: A: hypoventilation secondary to morbid obesity leads to hypoxemia, hypercarbia, and polycythemia. B: an autoimmune disorder causes destruction of the myelin and nerve axons within the brain and spinal cord. C: antibodies prevent acetylcholine from reaching the muscles by blocking or damaging the receptor sites. D: voluntary muscles are poorly controlled secondary to developmental brain defects that occur in utero.

*A: hypoventilation secondary to morbid obesity leads to hypoxemia, hypercarbia, and polycythemia* Reason: Some morbidly obese patients suffer from a condition called Pickwickian syndrome. When they lie in a supine position, excess adipose (fat) tissue on their chest wall impedes the mechanics of breathing, resulting in hypoventilation or apnea. As a result, hypoxemia and hypercarbia occur. Excess red blood cell production (polycythemia) occurs in response to hypoxemia. The patient commonly presents with a red face (due to polycythemia) and complains of daytime sleepiness. Multiple sclerosis (MS) is a chronic disease of the CNS caused by an autoimmune disorder that destroys myelin and nerve axons within the brain and spinal cord. MS causes random areas of myelin to become inflamed, detach from the axon, and self-destruct. The areas of destruction become sclerosed, hence the name multiple (many) sclerosis (to harden). Cerebral palsy (CP) is a nonprogressive, bilateral neuromuscular disorder in which voluntary muscles are poorly controlled. It results from developmental brain defects in utero, brain trauma at birth or in early childhood, or postpartum CNS infections. Patients with CP often have spastic movements of their limbs and poor posture; this impairs their ability to move in a controlled manner. Myasthenia gravis is characterized by chronic fatigability and muscle weakness, especially in the face and throat. In myasthenia gravis, antibodies prevent acetylcholine from reaching the muscles by blocking or damaging the receptor sites. This interruption in communication results in acute bouts of muscle weakness, usually during activity; the condition often improves with rest.

Polycythemia is a condition that results in: A: increased oxygen-carrying capacity of the blood. B: a marked increase in the core body temperature. C: lactic acidosis due to anaerobic metabolism. D: spontaneous bleeding due to a low platelet count.

*A: increased oxygen-carrying capacity of the blood* Reason: Polycythemia is defined as an increase in red blood cell production. It often occurs in response to hypoxia, but may occur for other reasons. Patients with emphysema, for example, are often polycythemic in response to chronic hypoxia; this is why they are commonly referred to as "pink puffers." Because red blood cells attach to the hemoglobin molecule and carry oxygen, polycythemia increases the oxygen-carrying capacity and efficiency of the blood. Of course, how well the tissues actually get oxygenated depends on the oxyhemoglobin saturation. A marked increase in core body temperature results from environment factors (eg, heatstroke), inflammation, and infection. Pyrogens are fever-causing agents that are produced by the immune system. When an infectious or inflammatory process exists, the body produces excess levels of pyrogens, resulting in pyrexia (fever). Patients with thrombocytopenia—a reduction in platelets—commonly experience spontaneous bleeding. Furthermore, thrombocytopenic patients have impaired hemostasis; even minor internal bleeding can be extremely serious. Aerobic metabolism is the normal metabolic process; it produces carbon dioxide and water as its byproducts. In the absence of oxygen, the cells convert from aerobic to anaerobic metabolism and produce lactic acid.

Easy bruising, lymph node enlargement, and splenomegaly are clinical manifestations of: A: leukemia. B: anemia. C: polycythemia. D: lymphoma.

*A: leukemia* Reason: Leukemia is cancer of the blood, and is caused by an abnormal proliferation (production by multiplication) of leukocytes (white blood cells) in the bone marrow. Leukemic cells impair the normal production of red blood cells (RBCs), white blood cells (WBCs), and platelets (thrombocytes); this results in anemia, leukopenia (low WBC count), and easy bleeding due to thrombocytopenia (low platelet count). In leukemia, excessive white blood cells accumulate in major organs (ie, spleen, liver, brain, and lymph), causing them to become enlarged (ie, splenomegaly [enlarged spleen], adenopathy [enlarged lymph nodes], hepatomegaly [enlarged liver]). Other signs and symptoms of leukemia include bone pain (due to increased pressure in the medullary canal of the bone), fever, fatigue, night sweats, and weight loss.

Treatment for a patient who has a pulse and a documented core body temperature of 92.8°F includes: A: passive and active external rewarming. B: 20 mL/kg boluses of warm normal saline. C: passive rewarming only. D: prophylactic antidysrhythmic therapy.

*A: passive and active external rewarming* Reason: According to current emergency cardiac care (ECC) guidelines, a patient with mild hypothermia (CBT of 93.2°F to 96.8°F [34°C to 36°C]) should be treated with passive rewarming, which involves removing wet clothing, applying warm blankets, and allowing the patient's CBT to rise naturally. If the patient has moderate hypothermia (CBT of 86°F to 93.2°F [30°C to 34°C]), passive and active external rewarming should be performed. Active external rewarming involves the use of heating blankets, or radiant heat from hot packs placed in the groin, neck, and axillae. Active internal rewarming, including the administration of warm (109.4°F [43°C]) IV fluids, warm-water peritoneal lavage, and esophageal rewarming tubes, should be performed if the patient has severe hypothermia (CBT of less than 86°F [30°C]). In most cases, active internal (core) rewarming will take place at the hospital. Prophylactic antidysrhythmic therapy is not indicated for hypothermic patients.

A 42-year-old male presents with difficulty breathing, diffuse wheezing, urticaria, and a blood pressure of 74/44 mm Hg. His skin is cool and clammy. Given his clinical presentation, which of the following interventions would be the LEAST effective? A: Diphenhydramine, 25 mg IV B: Epinephrine, 2 µg/min IV C: 1 to 2 L saline bolus D: Epinephrine, 0.3 mg SC

*A: schizophrenia* Reason: Schizophrenia is a psychiatric illness marked by impairments in the perception or expression of reality. Signs include auditory hallucinations (eg, the "roommate"), paranoid or bizarre delusions (eg, you are not to be trusted), and disorganized speech and thinking. Antipsychotics such as Olanzapine (Zyprexa), risperidone (Risperdal) and clozapine (Clozaril) are commonly prescribed to treat schizophrenia. The patient's presentation is not consistent with bipolar disorder. Bipolar disorder is a biphasic psychiatric condition characterized by alternations of depression and mania. Manic patients are excessively elated, overly talkative, and have accelerated speech. Medications used to treat bipolar disorder include lithium, quetiapine fumarate (Seroquel) and aripiprazole (Abilify). Trichotillomania, the recurrent pulling out of one's own hair, is an impulse control disorder. Nothing in the scenario suggests this condition. Obsessive-compulsive disorder (OCD) is a psychiatric condition characterized by intrusive and unwanted thoughts or worries (obsessions), which the patient attempts to alleviate or eliminate by performing ritualistic acts (compulsions). Selective serotonin reuptake inhibitor (SSRI) drugs, such as fluoxetine (Prozac), sertraline (Zoloft), and paroxetine (Paxil), are commonly prescribed to patients with OCD. This patient's behavior is not consistent with OCD.

In addition to CPR, treatment for a patient in cardiac arrest with a core body temperature of 90°F includes: A: spacing epinephrine doses at longer than standard intervals. B: limiting defibrillation to one attempt only if V-Fib is present. C: avoiding the use of antidysrhythmics such as amiodarone. D: mild hyperventilation to facilitate the production of heat.

*A: spacing epinephrine doses at longer than standard intervals* Reason: According to current emergency cardiac care (ECC) guidelines, cardiac arrest in a patient with moderate hypothermia (core body temperature [CBT] between 86°F [30°C] and 93°F [34°C]) should be treated with CPR, a single defibrillation every 2 minutes for V-Fib or pulseless V-Tach, and cardiac medication administration as indicated. However, the medications should be spaced at longer than standard intervals. In moderate hypothermia, the patient's basal metabolic rate (BMR) is slow, thus the onset and duration of action of medications (ie, vasopressors, antidysrhythmics) will be longer. Hyperventilation should be avoided in any patient. Not only does it hyperinflate the lungs and impede venous return to the heart, it facilitates heat loss, not production. If the cardiac arrest patient is severely hypothermic (CBT < 86°F [30°C]), cardiac medications should be withheld, and defibrillation, if indicated, should be attempted one time only.

A 33-year-old female was stung by a scorpion 45 minutes ago. She is conscious and alert, and presents with diffuse urticaria and intense itching. She denies chest tightness or difficulty breathing. Her breath sounds are clear and equal bilaterally. Her BP is 134/82 mm Hg, pulse is 104 beats/min and strong, respirations are 16 breaths/min and unlabored, and her SpO2 is 95% on room air. The MOST appropriate treatment for this patient includes: A: supplemental oxygen, IV of an isotonic crystalloid, 25 mg diphenhydramine IV. B: oxygen at 2 L/min via nasal cannula, 0.3 mg epinephrine 1:1,000 SQ, IV of normal saline. C: an IV of an isotonic crystalloid, 0.1 mg epinephrine 1:10,000 IV, 25 mg diphenhydramine IV. D: assisted ventilation, IV of normal saline, 3 to 5 mL of epinephrine 1:10,000 IV.

*A: supplemental oxygen, IV of an isotonic crystalloid, 25 mg diphenhydramine IV* Reason: Your patient is experiencing an allergic reaction; she is not in anaphylactic shock. Anaphylaxis typically occurs within minutes following exposure to an allergen, although its onset can be delayed for as long as an hour. Because she is hemodynamically stable and is not wheezing, the only drug that is indicated for her is 25 to 50 mg of diphenhydramine (Benadryl)—an antihistamine. Give supplemental oxygen as needed to maintain her SpO2 above 94%. You should establish vascular access in case her condition worsens and she requires further drug therapy. Unless the patient with an allergic reaction is hypotensive or is wheezing, epinephrine is not indicated. Diphendydramine alone is often effective in terminating the allergic reaction.

A 59-year-old woman with a history of Grave's disease presents with an altered mental status. Her skin is hot to the touch and her pulse rate is 160 beats/min. These findings are MOST consistent with: A: thyrotoxic crisis. B: addisonian crisis. C: Cushing syndrome. D: myxedema.

*A: thyrotoxic crisis* Reason: Thyrotoxic crisis (thyroid storm, thyrotoxicosis) is a hypermetabolic clinical syndrome caused by critically high levels of the thyroid hormones T3 (triiodothyronine) and T4 (thyroxine). Signs and symptoms include high fever (as high as 105°F to 106°F [40.5°C to 41.1°C]), hypertension, and profound tachycardia. Untreated, it can lead to cardiac arrest. Thyrotoxic crisis may occur in conjunction with Grave's disease, the most severe and common cause of hyperthyroidism, or it may occur if a patient takes too much of their prescribed thyroid supplement (ie, levothyroxine). Advanced hypothyroidism (myxedema) is a hypometabolic clinical syndrome caused by a deficiency of T3 and T4; the patient's signs and symptoms are not consistent with myxedema. Addisonian crisis, an acute manifestation of adrenal insufficiency, typically occurs after the abrupt cessation of corticosteroid therapy (ie, prednisone, hydrocortisone); this would not explain the patient's hypermetabolic state. Cushing syndrome, caused by excessive cortisol production by the adrenal glands, can also cause a hypermetabolic state. However, given the patient's history of Grave's disease, this is less likely.

Which of the following statements regarding hypoglycemia is correct? A: Hypoglycemia typically occurs within 4 to 6 hours after inadvertently taking too much insulin. B: A patient with prolonged hypoglycemia may require more than one dose of IV dextrose. C: Most diabetic patients develop symptoms when their blood glucose level falls below 90 mg/dL. D: The skin of a hypoglycemic patient is typically warm and dry secondary to severe dehydration.

*B: A patient with prolonged hypoglycemia may require more than one dose of IV dextrose* Reason: 25 g (50 mL) of 50% dextrose (D50) is usually sufficient to restore adequate circulating blood glucose levels and improve the patient's condition. However, if the hypoglycemic event is severe or prolonged, the patient's response to treatment may be delayed, thus requiring additional dosing. It is important to assess the patient's blood glucose level (BGL) and mental status after administering dextrose; doing so will help determine if additional dextrose is needed. Normal serum glucose levels range from 70 to 120 mg/dL. Hypoglycemia following an inadvertent overdose of insulin typically develops within seconds to minutes. The point at which a hypoglycemic patient becomes symptomatic varies. Some patients may develop symptoms when their BGL falls below 70 or 80 mg/dL; others may not develop symptoms until their BGL falls below 40 or 50 mg/dL. Diabetic patients with severe hypoglycemia (insulin shock, hypoglycemic crisis) usually have cool, clammy skin. Patients with severe hyperglycemia (diabetic coma, hyperglycemic crisis) typically present with warm, dry skin; this is a manifestation of dehydration secondary to the osmotic diuresis associated with hyperglycemia.

Which of the following can MOST easily be assessed without talking to a patient who is experiencing a behavioral crisis? A: Orientation B: Affect C: Memory D: Thought

*B: Affect* Reason: Mood and affect can be assessed in a patient experiencing a behavioral crisis without talking to him or her. Mood can be objectively noted via the patient's body language to determine if it is euphoric, sad, or labile. Affect—the expression of inner feelings—can be assessed by noting if it seems appropriate to the situation, is animated, angry, flat, or withdrawn. Assessment of memory, thought, and orientation generally require you to ask the patient direct questions. Simply observing the patient and listening to him or her can yield a lot of information, such as the potential for violence (ie, increased tone of voice, clenched fists).

A 60-year-old woman, who has been taking high doses of prednisone for several months to treat her rheumatoid arthritis, presents with weakness and fatigue that has progressively worsened. On appearance, her face appears swollen. You should be MOST suspicious that this patient has: A: Adrenal insufficiency. B: Cushing's syndrome. C: Addison's disease. D: Grave's disease.

*B: Cushing's syndrome* Reason: Cushing's syndrome is caused by excess cortisol production by the adrenal glands or by excessive use of cortisol or similar steroid (glucocorticoid) hormones. Tumors of the pituitary gland or adrenal cortex can stimulate the production of cortisol, for example, and lead to Cushing's syndrome. Administration of large amounts of corticosteroid hormones (ie, prednisone, methylprednisolone, dexamethasone, hydrocortisone) to treat conditions such as rheumatoid arthritis, asthma, and systemic lupus can also cause Cushing's syndrome. Regardless of the cause, excess cortisol causes characteristic changes in many body systems. Protein synthesis is impaired so that body proteins are broken down, which leads to loss of muscle fibers with resultant muscle weakness. Common signs and symptoms of Cushing's disease include weakness and fatigue, depression and mood swings, darkening of the skin (acanthosis) on the neck, and weight gain—especially on the abdomen, face ("moon face"), neck, and upper back. Primary adrenal insufficiency—also known as Addison's disease—is caused by atrophy or destruction of the adrenal glands, leading to a deficiency of all the steroid hormones produced by these glands. Grave's disease, a disease process associated with hyperthyroidism, occurs when the thyroid gland produces excess thyroid hormones.

Which of the following is MOST indicative of an infectious or communicable disease? A: Vomiting and diarrhea for 3 days B: Persistent fever and night sweats C: Sore throat and nasal discharge D: Severe headache and photophobia

*B: Persistent fever and night sweats* Reason: A persistent fever and night sweats should alert the paramedic to the possibility of a communicable or infectious disease. HIV and tuberculosis both present with these symptoms. Vomiting, diarrhea, sore throat, and a headache could indicate a variety of illnesses, not all of which are communicable or infectious.

Following prolonged dehydration, a 67-year-old man presents with hypotension, tachycardia, and oliguria. He has no past medical history. Which of the following is the MOST likely cause of his condition? A: Chronic renal failure B: Prerenal acute renal failure C: Postrenal acute renal failure D: Intrarenal acute renal failure

*B: Prerenal acute renal failure* Reason: Acute renal failure (ARF) is a sudden decrease in glomerular filtration rate (GFR), which causes toxins to accumulate in the blood. ARF is classified into three types, based on the area in which the failure occurs: prerenal, intrarenal, and postrenal. Prerenal ARF is caused by hypoperfusion of the kidneys; not enough blood passes into the glomeruli for them to produce filtrate. The most common causes of prerenal ARF are hypovolemia (blood loss or severe dehydration), trauma, sepsis, shock, and heart failure. Intrarenal ARF involves damage to one of three areas in the kidney: the glomeruli capillaries and small blood vessels, the cells of the renal tubules, or the renal parenchyma. This type of ARF can be caused by immune-mediated diseases (ie, type I diabetes) or by certain medications. Postrenal ARF is caused by obstruction of urine flow from the kidneys. The source of this obstruction is often a blockage of the urethra by an enlarged prostate gland, renal calculi (kidney stones), or strictures. As a result, pressure in the nephrons is increased, which causes them to stop functioning. Chronic renal failure is progressive and irreversible inadequate kidney function caused by the permanent loss of nephrons. This disease develops over months or years. More than half of all cases are caused by systemic disease, such as hypertension or diabetes.

Which of the following is the BEST way to minimize the hypoxia that occurs in an apneic patient following submersion in water? A: Immediate tracheal intubation B: Rescue breathing while in the water C: Frequent oropharyngeal suctioning D: Prophylactic abdominal thrusts

*B: Rescue breathing while in the water* Reason: Personnel with specialized training and experience in water rescue should retrieve a patient from the water whenever possible. As soon you as you reach the victim and determine that he or she is not breathing, immediate rescue breathing (mouth-to-mouth or mouth-to-mask) is the best (and quickest) way to minimize tissue hypoxia. Once the patient is removed from the water, continue ventilations with a bag-mask device and consider tracheal intubation. A significant number of drowning victims regurgitate; suction the oropharynx as needed and consider inserting a gastric tube to decompress the stomach. Do NOT perform abdominal thrusts; this may displace water from the stomach into the lungs, increasing the risk of pneumonitis and subsequent lung infection. Abdominal thrusts are only indicated for patients with a solid foreign body airway obstruction.

A 39-year-old man reports nausea, lack of coordination, and frequently recurring headaches that have been getting progressively worse over the past 2 months. He denies any past medical history or head injury. On the basis of this presentation, which of the following should you suspect? A: Ruptured cerebral arterial aneurysm B: Space-occupying intracranial lesion C: Chronic epidural hematoma D: Acute subarachnoid hemorrhage

*B: Space-occupying intracranial lesion* Reason: The worsening and persistent symptomatology that this patient is experiencing suggests a space-occupying intracranial lesion (ie, a tumor). Ruptured cerebral aneurysms and acute epidural bleeds present with a sudden onset of symptoms. Because epidural hematomas are caused by an arterial hemorrhage, they are acute, not chronic. Subdural hematomas are caused by a venous hemorrhage; therefore, they can be acute or chronic.

During your assessment of a patient with a suspected neurologic disorder, you ask him to shrug his shoulders and turn his head from side to side. Which of the following cranial nerves are you assessing? A: Glossopharyngeal B: Spinal accessory C: Trigeminal D: Vestibulocochlear

*B: Spinal accessory* Reason: If time and patient condition permits, assessment of the 12 pairs of cranial nerves should be performed. The spinal accessory nerve (XI), a motor nerve, controls shoulder and neck movements. Asking the patient to shrug his shoulders and turn his head from side to side assesses cranial nerve XI. The trigeminal nerve (V) provides motor control to the muscles of chewing and sensory control to the face, sinuses, and teeth. Asking the patient to clench his teeth and then lightly stroking your finger over his forehead and cheeks and asking him to identify where he is being touched assesses cranial nerve V. The vestibulocochlear nerve (VIII), a sensory nerve, controls hearing and balance perception. Checking a patient's hearing and asking him to stand on one leg (if safe to do so) assesses cranial nerve VIII. The glossopharyngeal nerve (IX) provides motor control to the throat and swallowing mechanism and sensory control to the tongue, throat and ear. Asking the patient to swallow assesses cranial nerve IX. Refer to your paramedic textbook regarding assessment of all of the cranial nerves.

You are called to a residence for a woman who swallowed an unknown quantity of pills. Which of the following should you inquire about FIRST? A: A history of psychiatric care B: What kind of pills were taken C: The patient's weight in kilograms D: When the pills were taken

*B: What kind of pills were taken* Reason: After ensuring scene safety, your initial action when caring for any patient is to ensure a patent airway and adequate breathing. After doing this, your next action for this patient should be to determine what kind of pills she took. Then determine how long ago she took the pills. This information will enable you to provide the most appropriate treatment, which may include an antedote. Determining the patient's weight in kilograms is also important as many antedotes are weight-based. When you have determined what was taken, when it was taken, and the patient's weight in kilograms, contact medical control or the poison control center. Whether or not the patient has a psychiatric history will not influence the immediate care that you provide, although you should inquire about this at some point.

A man was trapped in his burning house for approximately 20 minutes before being rescued by firefighters. Assuming that he was exposed to carbon monoxide (CO), cyanide, or both, it is important for the paramedic to remember that: A: based on the length of exposure, the patient's carboxyhemoglobin level is at least 40% to 50%. B: carbon monoxide and cyanide are chemical asphyxiants that displace oxygen and impair tissue oxygenation. C: cyanide is typically found in higher amounts during structural fires than carbon monoxide. D: unless the patient was unconscious while he was trapped, significant CO or cyanide toxicity is unlikely.

*B: carbon monoxide and cyanide are chemical asphyxiants that displace oxygen and impair tissue oxygenation* Reason: Carbon monoxide (CO) and hydrogen cyanide, both of which can be found in significant concentrations in a burning structure, are chemical asphyxiants that displace oxygen in room air. Although their mechanisms of action are different, both chemicals impair tissue oxygenation and cause significant tissue hypoxia. Significant CO and cyanide toxicity should be suspected in any patient who is trapped in an enclosed environment—regardless of whether a loss of consciousness occurred and especially if fire was involved. Blood levels of carboxyhemoglobin (COHb)—a toxic compound that is formed when CO attaches to hemoglobin molecules—and cyanide must be quantified by arterial blood gas and other lab analysis, although portable CO monitors (CO-oximeters) are available for use in the prehospital setting.

Shortly after his dialysis treatment, a 66-year-old man presents with confusion, a headache, and nausea. You should suspect: A: interstitial nephritis. B: disequilibrium syndrome. C: severe hyperkalemia. D: acute air embolism.

*B: disequilibrium syndrome* Reason: Dialysis rapidly lowers the concentration of urea in the blood, whereas the concentration of solutes in the cerebrospinal fluid (CSF) remains high. Water moves by osmosis from a solution of lower concentration into a solution of higher concentration. Thus, as a consequence of dialysis, water initially shifts from the bloodstream into the CSF, which mildly increases intracranial pressure. If this occurs, the patient may experience disequilibrium syndrome, a condition characterized by nausea, vomiting, headache, and confusion. After a few hours, the fluid will re-equilibrate between the blood and CSF, and the patient's symptoms will resolve on their own. Interstitial nephritis, a cause of intrarenal acute renal failure, is caused by chronic inflammation of the interstitial cells surrounding the nephrons; this condition would not explain the patient's symptoms. Acute air embolism may occur if air enters any of the fittings or connections in the dialysis system; signs and symptoms include acute dyspnea, hypotension, and cyanosis. Hyperkalemia, an increase in serum potassium, would most likely occur if a patient missed a dialysis treatment; signs and symptoms include profound muscle weakness, QRS widening, and peaked T waves.

You are assessing a 32-year-old female who complains of chills, muscle aches, and a headache. She tells you that she began feeling bad a few days ago, and has been taking ibuprofen for her headache. Her blood pressure is 130/72 mm Hg, pulse is 118 beats/min and strong, and respirations are 16 breaths/min and unlabored. The patient denies any significant medical problems and is conscious and alert. Further assessment of this patient will MOST likely reveal: A: hypoglycemia. B: fever. C: petechiae. D: wheezing.

*B: fever* Reason: This patient's clinical presentation is consistent with infection with the influenza virus—she probably has the flu. Chills are generally an indication of fever, and febrile patients are typically tachycardic. In addition to fever and chills, common signs and symptoms of the flu include muscle aches, anorexia, headache, and malaise. These symptoms are often followed by an upper respiratory infection and cough that may last for up to 7 days. Elderly and immunocompromised patients are at greater risk for secondary bacterial infections—for example, pneumonia—in which case localized wheezing may be observed. Because of her mental status and absence of any significant medical problems, hypoglycemia is unlikely, although it would not be unreasonable to assess her blood glucose level. A rash (ie, purpura, petechiae) is common in patients with severe sepsis; although this patient is ill, she is not septic.

Law enforcement calls you to a nearby alley, where they found an unresponsive young female. One of the officers states that the patient is a known drug abuser. The patient has slow, shallow breathing; a slow, weak pulse; and bilaterally constricted pupils. As your partner begins assisting her ventilations, you apply the cardiac monitor, which reveals sinus bradycardia. After establishing vascular access, you should: A: begin cardiac pacing at once. B: give 0.4 to 2 mg of naloxone. C: give a 500-mL fluid bolus. D: give 0.5 mg of atropine.

*B: give 0.4 to 2 mg of naloxone* Reason: The patient has signs of CNS depression, which is likely the result of an opiate (narcotic) overdose (unresponsiveness, hypoventilation, constricted [pinpoint] pupils; bradycardia). The most immediate priority is to ensure that her airway is patent and begin assisting her ventilations. After establishing vascular access (IV or IO), the initial drug of choice for this patient would be naloxone (Narcan) in a dose of 0.4 to 2 mg. Because of her history or drug abuse, you should start with 0.4 mg increments of naloxone. Titrate the naloxone to the patient's respirations, heart rate, and BP. If her clinical status is refractory to naloxone, consider atropine or transcutaneous cardiac pacing (TCP). The patient's clinical presentation is caused by CNS depression, not hypovolemia, so IV fluid boluses will likely be ineffective.

A 46-year-old woman was found unresponsive on her couch by a concerned neighbor. According to the neighbor, the patient uses heroin, but recently boasted that she has "been clean." The patient is unresponsive; has rapid, shallow breathing; and slow, weak radial pulses. You should: A: provide some form of positive-pressure ventilation, apply the cardiac monitor, establish vascular access, and administer 25 g of 50% dextrose IV push. B: insert an oral airway, assist her ventilations with a bag-mask device, apply the cardiac monitor, consider transcutaneous pacing, and establish vascular access. C: apply oxygen via nonrebreathing mask, apply the cardiac monitor, establish vascular access, and administer 2 mg of naloxone slow IV push. D: secure her airway with an endotracheal tube, apply the cardiac monitor, establish vascular access, and administer 1 mg of atropine rapid IV push.

*B: insert an oral airway, assist her ventilations with a bag-mask device, apply the cardiac monitor, consider transcutaneous pacing, and establish vascular access* Reason: This patient's clinical presentation—specifically her rapid, shallow respirations—are not consistent with a heroin overdose. As with all narcotic overdoses, heroin causes hypoventilation (ie, slow, shallow breathing) due to its central nervous system depressant effects. Your first priority is to establish a patent airway, insert an airway adjunct to help maintain her airway, and begin assisting her ventilations. Rapid, shallow (reduced tidal volume) respirations often do not produce adequate minute volume and should be treated with positive-pressure ventilation, not a nonrebreathing mask. Transcutaneous cardiac pacing (TCP) is an acceptable intervention for any patient with unstable bradycardia and should be initiated without delay. Atropine is also an appropriate intervention; however, the correct dose for bradycardia is 0.5 mg. If TCP and atropine are ineffective, consider the possibility of a coingestion (ie, heroin and a tricyclic antidepressant). Given the patient's history of heroin use, it would clearly be appropriate to administer naloxone; however, based on her initial presentation, it would not be the initial intervention of choice. Dextrose would be appropriate if hypoglycemia is documented by glucometer.

A 70-year-old male presents with confusion, a blood pressure of 74/56 mm Hg, and a pulse of 80 beats/min and weak. His son tells you that he has had black tarry stools for the past 3 days, but refused to go to the hospital. He further tells you that his father has hypertension and Alzheimer's disease, and hands you a medication list that includes Toprol, Zestril, Zyprexa, and vitamins. You should be MOST concerned that this patient: A: has GI bleeding. B: is hypoperfused. C: is hypoglycemic. D: is not tachycardic.

*B: is hypoperfused* Reason: The most immediate threat to this patient's life is shock (hypoperfusion); therefore, your most immediate concern should focus on taking actions to improve oxygenation and perfusion (eg, high-flow oxygen, IV fluid boluses). The presence of dark, tarry stools (melena) clearly indicates gastrointestinal (GI) bleeding; however, the most urgent issue is not the source of the bleeding (you can't stop it anyway)—it is the fact that the bleeding has caused inadequate perfusion to the vital organs of his body. The patient's confusion could be the result of his Alzheimer's disease, inadequate cerebral perfusion secondary to hypovolemia, or both. Although he does not have a history of diabetes, his blood glucose level (BGL) should be assessed to rule out hypoglycemia as a contributing cause to his confusion. The absence of tachycardia in this patient, which is impairing his ability to compensate for decreased cardiac output, is likely the result of two factors: age-related deterioration of the sympathetic nervous system with reduced catecholamine production, and the fact that he is taking a beta-blocker medication (metoprolol [Toprol]). Beta-blockers are used to treat hypertension and certain types of tachydysrhythmias; they reduce heart rate, myocardial contractility, and blood pressure secondary to sympathetic nervous system blockade.

Easy bruising, lymph node enlargement, and splenomegaly are clinical manifestations of: A: anemia. B: leukemia. C: lymphoma. D: polycythemia.

*B: leukemia* Reason: Leukemia is cancer of the blood, and is caused by an abnormal proliferation (production by multiplication) of leukocytes (white blood cells) in the bone marrow. Leukemic cells impair the normal production of red blood cells (RBCs), white blood cells (WBCs), and platelets (thrombocytes); this results in anemia, leukopenia (low WBC count), and easy bleeding due to thrombocytopenia (low platelet count). In leukemia, excessive white blood cells accumulate in major organs (ie, spleen, liver, brain, and lymph), causing them to become enlarged (ie, splenomegaly [enlarged spleen], adenopathy [enlarged lymph nodes], hepatomegaly [enlarged liver]). Other signs and symptoms of leukemia include bone pain (due to increased pressure in the medullary canal of the bone), fever, fatigue, night sweats, and weight loss.

A 29-year-old male ingested a bottle of extra strength acetaminophen. He presents with malaise and palpable tenderness to the right upper quadrant of his abdomen. He is conscious and alert, his BP is 136/66 mm Hg, his pulse is 100 beats/min and regular, his respirations are 16 breaths/min, and his oxygen saturation is 98%. The MOST appropriate treatment for this patient includes: A: assisted ventilation, IV access, a 500 mL normal saline bolus, 1 mEq/kg of sodium bicarbonate, cardiac monitoring, and transport. B: oxygen via nasal cannula at 2 to 3 L/min, an IV of normal saline set to keep the vein open, cardiac monitoring, and transport. C: oxygen via nasal cannula at 4 to 6 L/min, 1 to 2 g/kg of activated charcoal, IV fluid boluses, cardiac monitoring, and transport. D: oxygen via nonrebreathing mask, 30 mL of ipecac, an IV of normal saline set to keep the vein open, cardiac monitoring, and transport.

*B: oxygen via nasal cannula at 2 to 3 L/min, an IV of normal saline set to keep the vein open, cardiac monitoring, and transport* Reason: Acetaminophen (APAP), the active ingredient in Tylenol, is rapidly absorbed from the GI tract, producing peak serum levels in 30 to 120 minutes. Signs of toxicity appear in four distinct stages. Stage I (ingestion < 24 hours ago) symptoms include nausea, vomiting, anorexia, pallor, and malaise. Stage II (24 to 72 hours) is marked by right upper quadrant pain and palpable abdominal tenderness. Stage III (72 to 96 hours) is marked by metabolic acidosis, renal failure, coagulopathies, and recurring GI symptoms. During stage IV (> 96 hours), the patient either slowly recovers or dies of liver failure. Your patient's signs are consistent with stage II. Based on his clinical presentation, treatment should include supplemental oxygen (his breathing is adequate and his SpO2 is 98%; a nasal cannula is sufficient), an IV set to keep the vein open (his BP [136/66 mm Hg] does not warrant fluid boluses), continuous ECG monitoring, and transport. Activated charcoal would be of no benefit; the drug is no longer in the stomach, it is systemic. Due to the risk of aspiration, do not induce vomiting with syrup of ipecac for any ingestion. Sodium bicarbonate, if indicated, should be guided by arterial blood gas analysis. The antidote for APAP toxicity is acetylcysteine (Acetadote); however, it is given at the hospital.

A 26-year-old woman was bitten on the leg by a rattlesnake while hiking. She is conscious and alert, but is anxious. Her BP is 114/66 mm Hg, her heart rate is 112 beats/min, her respirations are 20 breaths/min, and her oxygen saturation is 97%. Treatment for her should include: A: oxygen via non-rebreathing mask, elevating the extremity above the level of the heart, and a 20 mL/kg IV fluid bolus. B: oxygen via nasal cannula, splinting the affected extremity, and an IV line of normal saline set at a keep-vein-open rate. C: oxygen via non-rebreathing mask, applying a chemical ice pack to the bite wound, and splinting the affected extremity. D: oxygen via nasal cannula, elevating the extremity above the level of the heart, and 1 gm of calcium chloride IV push.

*B: oxygen via nasal cannula, splinting the affected extremity, and an IV line of normal saline set at a keep-vein-open rate* Reason: Treatment for a pit viper bite includes placing the patient in a comfortable position, administering supplemental oxygen (given her oxygen saturation [97%], a nasal cannula is appropriate for this patient), splinting the affected extremity and keeping it below the level of the heart, and promptly transporting to an appropriate facility. Monitor the patient's cardiac rhythm and establish an IV line. Fluid boluses are not indicated for this patient because her BP is stable; set the IV at a keep-vein-open (KVO) rate. Calcium chloride is not indicated for snakebites. Do NOT apply ice to a snakebite; doing so causes localized vasoconstriction; if envenomation occurred, this could push the venom further into the circulation. Alert the receiving facility early and be prepared to provide cardiopulmonary support (ie, bag-mask ventilation, IV fluid boluses) if the patient's clinical status deteriorates.

After performing your primary assessment and initial treatment of an unresponsive patient who has not been injured, you should: A: obtain a SAMPLE history. B: perform a rapid assessment. C: obtain vital signs. D: perform a detailed head-to-toe exam.

*B: perform a rapid assessment* Reason: When the primary assessment and initial treatment of an unresponsive medical patient is complete, the next step is to perform a rapid scan of the patient's body, which involves a quick head-to-toe assessment. The purpose of the rapid body scan is to further assess for and treat life-threats not detected in the primary assessment. If the patient is responsive, perform a secondary assessment, which should focus on his or her chief complaint. At this point, you should also obtain vital signs and a SAMPLE history.

Functions of the hypothalamus include: A: maintaining equilibrium and balance. B: regulating appetite. C: influencing breathing. D: controlling level of awareness.

*B: regulating appetite* Reason: The "hunger" center of the hypothalamus promotes eating. Among many other functions, the hypothalamus regulates body temperature, assists in the regulatory control of the pituitary gland, and promotes urine release from the bladder. The influence of breathing are functions of the pons and medulla, which are a part of the brainstem. Maintenance of equilibrium and balance are functions of the cerebellum. Control of emotions and level of awareness are functions of the cerebrum, which is the largest portion of the brain (also referred to as the gray mater).

A 64-year-old man presents with an acute onset of left-sided weakness, right-sided facial droop, and slurred speech. He is conscious, but confused. You should be MOST suspicious for: A: right-sided hemorrhagic stroke. B: right-sided ischemic stroke. C: left-sided ischemic stroke. D: left-sided hemorrhagic stroke.

*B: right-sided ischemic stroke* Reason: Recalling that the right side of the brain controls the left side of the body and vice versa, this patient's clinical presentation is most consistent with an ischemic stroke to the right cerebral hemisphere. Ischemic strokes, caused by a blocked cerebral artery, typically present with confusion, unilateral weakness (hemiparesis) or paralysis (hemiplegia) on the opposite (contralateral) side of the stroke, a facial droop on the same (ipsilateral) side as the stroke, and dysarthria (slurred or poorly articulated speech). Pupillary abnormalities, if observed, typically occur on the ipsilateral side because of optic nerve crossover in the brain. The patient's clinical presentation is less consistent with a hemorrhagic stroke, which typically presents with a sudden, severe headache; a rapid loss of consciousness; and signs of increased intracranial pressure (ie, hypertension, bradycardia, breathing abnormalities). A hemorrhagic stroke is the result of a ruptured cerebral artery (ie, an aneurysm).

You are assessing a patient who presents with a melena and abdominal pain. Which of the following medical history findings is the MOST clinically significant? A: Hypertension B: von Willebrand disease C: Type II diabetes mellitus D: Hashimoto disease

*B: von Willebrand disease* Reason: All of the diseases listed are clinically significant. However, the presence of von Willebrand disease in a patient with internal or external bleeding should concern you the most. Von Willebrand disease, a bleeding disorder similar to hemophilia, is caused by a deficiency of von Willebrand factor (vWF), a blood clotting protein. vWF circulates attached to factor VIII, another blood clotting protein. As a consequence of von Willebrand disease, the normal process of hemostasis is interrupted and the patient will continue to bleed. Hashimoto disease, also called Hashimoto thyroiditis, is a cause of hyperthyroidism; it is an autoimmune disorder that affects the thyroid-stimulating hormone (TSH) receptors. While hypertension, Hashimoto disease, and type II diabetes can certainly complicate the clinical picture of any sick or injured patient, bleeding disorders in a patient with active bleeding would clearly be the most detrimental.

A 43-year-old woman was stung by a scorpion. Within 5 minutes, she developed swelling to her face and diffuse urticaria. She is confused and has a BP of 80/60 mm Hg. After administering supplemental oxygen, you should give: A: 125 mg of methylprednisolone IV. B: 25 to 50 mg of diphenhydramine IM. C: 0.1 mg of epinephrine 1:10,000 IV. D: a rapid 500 mL normal saline bolus.

*C: 0.1 mg of epinephrine 1:10,000 IV*

A 43-year-old woman was stung by a scorpion. Within 5 minutes, she developed swelling to her face and diffuse urticaria. She is confused and has a BP of 80/60 mm Hg. After administering supplemental oxygen, you should give: A: 125 mg of methylprednisolone IV. B: 25 to 50 mg of diphenhydramine IM. C: 0.1 mg of epinephrine 1:10,000 IV. D: a rapid 500 mL normal saline bolus.

*C: 0.1 mg of epinephrine 1:10,000 IV* Reason: The patient is in anaphylactic shock; she has an altered mental status, is hypotensive, has diffuse urticaria (hives), and facial swelling (angioedema). After ensuring a patent airway and improving ventilation and oxygenation, the most crucial intervention is epinephrine. For anaphylactic shock, the adult dose is 0.1 mg (1 mL) of a 1:10,000 solution IV. Through its alpha-1 and beta-2 adrenergic effects of vasoconstriction and bronchodilation, epinephrine reverses the hypotension and bronchospasm associated with anaphylactic shock. For refractory hypotension, administer fluid boluses and consider an epinephrine infusion. Diphenhydramine (Benadryl) is given after epinephrine to block the release of histamines that are causing the allergic reaction; the adult dose is 25 to 50 mg IV. Methylprednisolone (Solu-Medrol), a glucocorticoid (steroid) anti-inflammatory drug, may be given as an adjunct to epinephrine and diphendydramine; the adult dose is 125 mg.

A 59-year-old male with a history of hypertension and diabetes presents with dark, tarry stools. He is confused, has a blood pressure of 84/62 mm Hg, and a pulse rate of 74 beats/min and weak. Which of the following would MOST likely explain his heart rate? A: Increased parasympathetic tone in response to hypovolemia B: Insulin suppressing the patient's sympathetic nervous system C: A prescribed adrenergic blocking agent to treat his hypertension D: Inadvertent overdose of his prescribed hypoglycemic medication

*C: A prescribed adrenergic blocking agent to treat his hypertension* Reason: Adrenergic blocking medications, such as beta blockers, are commonly used to treat hypertension. Examples of beta blockers include propranolol (Inderal), metaprolol (Lopressor), atenolol (Tenormin), and labetalol (Normodyne). Beta blockers suppress the sympathetic nervous system, which reduces heart rate and myocardial contractility. The patient in this scenario is in shock due to gastrointestinal bleeding. His heart rate—which you would expect to be fast—is in a normal range. This is most likely the result of his prescribed beta blocker medication, which is blunting the sympathetic nervous system's compensatory response to shock. In hypovolemic shock, the sympathetic nervous system—not the parasympathetic nervous system—is stimulated, resulting in tachycardia, pallor, and diaphoresis, among other signs.

A 60-year-old male with chronic alcoholism presents with an acute onset of hematemesis. His blood pressure is 80/40 mm Hg, pulse is 130 beats/min and weak, and respirations are 28 breaths/min and shallow. What pathophysiologic process is MOST likely responsible for his condition? A: Acute rupture of an esophageal artery secondary to repeated episodes of vomiting B: Severe bleeding from gastric ulcers that formed due to the excessive intake of alcohol C: Abnormally enlarged esophageal veins secondary to impaired blood flow to the liver D: Erosion of the large esophageal blood vessels caused by the toxic effects of alcohol

*C: Abnormally enlarged esophageal veins secondary to impaired blood flow to the liver* Reason: This patient likely has ruptured esophageal varices. Esophageal varices are a complication of portal hypertension (increased pressure in the portal vein [the vein that carries blood from the intestines to the liver]) due to liver disease. Liver disease (ie, hepatitis, cirrhosis) is common in patients with chronic alcoholism. As blood flow to the diseased liver is blocked, blood backs up into the smaller, more fragile blood vessels in the lower part of the esophagus. As a result, these esophageal vessels become abnormally enlarged (varices). Esophageal varices are generally asymptomatic unless they rupture, in which case life-threatening hemorrhage can occur. Other complications associated with chronic alcoholism include impaired blood-clotting mechanisms, hypoglycemia, and gastritis due to the irritant effect of alcohol on the gastric lining. Repeated episodes of vomiting could rupture an esophageal vessel and cause hematemesis; however, there is no indication that this patient has had repeated episodes of vomiting.

Which of the following can MOST easily be assessed without talking to a patient who is experiencing a behavioral crisis? A: Thought B: Memory C: Affect D: Orientation

*C: Affect* Reason: Mood and affect can be assessed in a patient experiencing a behavioral crisis without talking to him or her. Mood can be objectively noted via the patient's body language to determine if it is euphoric, sad, or labile. Affect—the expression of inner feelings—can be assessed by noting if it seems appropriate to the situation, is animated, angry, flat, or withdrawn. Assessment of memory, thought, and orientation generally require you to ask the patient direct questions. Simply observing the patient and listening to him or her can yield a lot of information, such as the potential for violence (ie, increased tone of voice, clenched fists).

Which of the following hormones stimulates the kidneys to reabsorb sodium and excrete potassium? A: Adrenocorticotropic hormone B: Antidiuretic hormone C: Aldosterone D: Somatostatin

*C: Aldosterone* Reason: If the body experiences a drop in blood pressure or volume, a decrease in sodium levels, or an increase in potassium levels, the adrenal cortex is stimulated to secrete aldosterone (a mineralocorticoid). Aldosterone stimulates the kidneys to reabsorb sodium from the urine and excrete potassium by altering the osmotic gradient in the blood. When sodium is reabsorbed into the blood, water follows; this action increases both blood volume and pressure. Somotostatin is a hormone secreted by the delta cells of the pancreas; it inhibits insulin and glucagon secretion. Antidiuretic hormone (ADH), which is released by the pituitary gland, regulates water balance in the body. If a person is dehydrated, ADH secretion increases, which stimulates the renal tubules to reabsorb sodium and water and inhibits diuresis. If a person is overhydrated, ADH secretion decreases, thereby facilitating diuresis. Adrenocorticotropic hormone (ACTH) is secreted by the pituitary gland; it stimulates the adrenal cortex to manufacture and secrete cortisol.

A 27-year-old female overdosed on an unknown type of drug. Her skin is hot and flushed, her breathing is rapid and deep, and she has an acetone odor on her breath. Her BP is 98/64 mm Hg and her heart rate is 120 beats/min. Which of the following drugs would MOST likely explain her clinical presentation? A: Alcohol B: Heroin C: Aspirin D: Cocaine

*C: Aspirin* Reason: Of the drugs listed, aspirin (acetylsalicylic acid [ASA]) overdose would be the most likely cause of the patient's clinical presentation. Aspirin toxicity causes significant metabolic acidosis; the respiratory buffer system responds by increasing the rate and depth of breathing, resulting in a compensatory respiratory alkalosis. Hyperthermia (as evidenced by hot, flushed skin) is often associated with aspirin toxicity. Although an acetone breath odor is classically associated with hyperglycemic ketoacidosis, methyl alcohol, isopropyl alcohol, and aspirin toxicity can also cause an acetone breath odor. Heroin overdose is clearly unlikely. Heroin is a narcotic CNS depressant; overdose would result in slow, shallow breathing; hypotension, and bradycardia. Alcohol is also a CNS depressant; although the intoxicated patient can present with a breath odor that could be mistaken for ketoacidosis, severe toxicity causes respiratory depression. An overdose of cocaine, a sympathomimetic drug, can also cause hyperthermia; however, you would expect the patient to be significantly hypertensive and tachycardic.

A 23-year-old male is found unresponsive. According to a friend, the patient had a headache and said that he was going to take a nap. His breathing is rapid and shallow, his pulse is rapid and weak, and he is profusely diaphoretic. Which of the following represents the MOST appropriate treatment for this patient? A: Preoxygenate him and prepare to intubate, start an IV line, administer 2 mg of naloxone, assess his vital signs, and apply the cardiac monitor. B: Hyperventilate him with a bag-mask device, conduct an in-depth neurologic exam, obtain a 12-lead ECG, and administer 1 mg of glucagon IM. C: Assist ventilations with a bag-mask device, assess his blood glucose level, apply the cardiac monitor, obtain vital signs, and establish vascular access. D: Apply oxygen via nonrebreathing mask, start an IV line, administer 25 g of 50% dextrose, apply a cardiac monitor, and obtain vital signs.

*C: Assist ventilations with a bag-mask device, assess his blood glucose level, apply the cardiac monitor, obtain vital signs, and establish vascular access* Reason: The patient's respirations are likely not producing adequate minute volume and should be assisted with a bag-mask device. Hyperventilation should be avoided; it increases the incidence of gastric distention and may impair venous return to the heart due to increased intrathoracic pressure. You must next try to determine the cause of his unresponsiveness. Tachypnea and tachycardia are not consistent with a narcotic overdose; therefore, naloxone (Narcan) will likely have no effect. He may be hypoglycemic; however, before administering dextrose or glucagon, assess his blood glucose level first. Cardiac monitoring is indicated in order to detect life-threatening dysrhythmias; obtain a 12-lead ECG when possible. Vascular access is indicated in case fluid boluses or drug therapy is necessary. Intubation may be necessary if the patient requires prolonged ventilatory support or if you are unable to effectively ventilate with a bag-mask device. The ability to rule out (or in) causes of altered mental status in the field is relatively limited; therefore, rapid transport is essential.

You are called to the residence of an elderly man whose daughter states that he is not acting right. The patient becomes combative when you attempt to assess him. He refuses supplemental oxygen and states that you are not taking him anywhere. What is the MOST appropriate course of action? A: Administer 5 mg diazepam IM to calm and sedate him. B: Start an IV line and administer 25 gm of 50% dextrose. C: Calmly talk to him and try to obtain a glucose reading. D: Gently restrain him and transport him to the hospital.

*C: Calmly talk to him and try to obtain a glucose reading*

You are called to a residence for a 39-year-old woman, who, according to her husband, is "not acting right." She is confused, is experiencing hallucinations, and is repetitively smacking her lips. Which of the following should you suspect? A: Generalized seizure B: Focal motor seizure C: Complex partial seizure D: Simple partial seizure

*C: Complex partial seizure* Reason: Seizures are classified as being generalized or partial. Your patient's presentation is consistent with a complex partial seizure. Partial seizures affect a limited part of the brain and are further divided into simple partial and complex partial. Simple partial seizures involve movement (frontal lobe) or sensations (parietal lobe) to one part of the body. A focal motor seizure is a simple partial seizure with localized motor activity. There may be spasm or clonus (jerking) of one muscle or muscle group, which may remain localized or may spread to adjacent muscles (Jacksonian march). Complex partial seizures involve changes in level of consciousness. The patient can become confused, lose alertness, experience hallucinations, or may be unable to speak. Automatisms, such as lip smacking, chewing, swallowing, may occur with complex partial seizures. Generalized seizures affect the entire brain. Tonic/clonic seizures (full body jerking movements), absence seizures (freezing or staring), and pseudoseizures (tonic/clonic, but caused by a psychiatric mechanism) are examples of generalized seizures.

When assessing a patient who was stung by a bee, which of the following assessment findings is MOST indicative of anaphylactic shock? A: A fine, red rash B: Known allergy to bees C: Diaphoretic skin D: Flushing of the skin

*C: Diaphoretic skin* Reason: Allergic reactions can be mild, moderate, or severe. General signs of an allergic reaction include a fine red rash or hives (urticaria), flushed skin, itching or burning of the skin (pruritis), and watery eyes. In a mild or moderate allergic reaction, respiratory and circulatory functions are generally not affected. If the histamine release is overwhelming, however, the patient develops anaphylactic shock, which is characterized by general signs of an allergic reaction plus respiratory and circulatory compromise. Diaphoresis, a sign of shock, is not typically observed in patients with mild or moderate allergic reactions because systemic perfusion is not compromised. A known allergic history is not, in and of itself, an indicator of anaphylactic shock; however, it should raise your concern that an allergic reaction may occur.

In addition to CPR, treatment for a patient in cardiac arrest with a core body temperature of 90°F includes: A: mild hyperventilation to facilitate the production of heat. B: spacing epinephrine doses at longer than standard intervals. C: limiting defibrillation to one attempt only if V-Fib is present. D: avoiding the use of antidysrhythmics such as amiodarone.

*C: Epinephrine, 0.3 mg SC* Reason: The patient in this scenario is experiencing signs and symptoms of anaphylactic shock—dyspnea, wheezing, urticaria (hives). Furthermore, the fact that he is hypotensive indicates that he is in decompensated shock. Poor peripheral perfusion is evidenced by his cool, clammy skin; blood has been shunted away from the skin to the vital organs. Compared to the intravenous (IV) and intraosseous (IO) routes, medications given via the subcutaneous (SC) and intramuscular (IM) routes take longer to absorb into the central circulation—even in patients with adequate peripheral perfusion. Therefore, it would clearly stand to reason that a medication administered via the SC route to a patient with poor peripheral perfusion would have an even more delayed effect. For patients with anaphylactic shock, epinephrine should be administered via the IV or IO route. The appropriate dose is 0.1 mg (1 mL) of a 1:10,000 solution. An IV infusion of epinephrine at 1 to 4 µg/min also would be appropriate, and may avoid the need to give repeated epinephrine boluses for severely hypotensive patients. If hypotension does not respond rapidly to epinephrine, crystalloid fluid boluses should be given; as much as 1 to 2 L may be needed initially. Diphenhydramine (Benadryl), 25 to 50 mg IV, is given after epinephrine; it blocks the release of histamines that are causing the allergic reaction. Patients with mild or moderate allergic reactions who have adequate peripheral perfusion are commonly given epinephrine 1:1,000 via the SC or IM route in a dose of 0.3 to 0.5 mg (0.3 to 0.5 mL). Benadryl can also be given via the IM route.

Which of the following clinical presentations is consistent with significant insecticide exposure? A: A dry cough, tachycardia, and hypertension B: Acute urinary retention and abdominal pain C: Excessive salivation and severe bradycardia D: Extreme hyperactivity and pupillary dilation

*C: Excessive salivation and severe bradycardia* Reason: Organophosphates, such as what is found in pesticides/insecticides and chemical nerve agents (VX, sarin, tabun, soman), deactivate acetylcholinesterase (AChE), an enzyme that regulates the degradation of acetylcholine (ACh). ACh is the chemical neurotransmitter of the parasympathetic nervous system. Without AChE, there is nothing to regulate ACh degradation; this would cause cardiovascular collapse secondary to massive parasympathetic stimulation. The clinical presentation of severe organophosphate toxicity can be recalled using the mnemonic DUMBELS, which stands for defecation, urination, miosis (pupillary constriction), bronchorrhea and bradycardia, emesis, lacrimation, and salivation. Treatment includes atropine sulfate, which blocks the effects of ACh, and pralidoxime chloride (2-PAM, Protopam), which reactivates AChE. A commercial auto-injector (DuoDote) is available; it contains both pralidoxime and atropine.

Obsessive-compulsive disorder falls under which of the following categories? A: Schizophrenia B: Posttraumatic stress disorder C: Generalized anxiety disorder D: Organic brain syndrome

*C: Generalized anxiety disorder* Reason: Generalized anxiety disorder includes a broad spectrum of psychiatric disorders including obsessive-compulsive disorder (OCD), panic attacks, and social phobias such as agoraphobia (fear of the market place). Schizophrenia is a severe psychiatric illness in which the patient has lost touch with reality; it is often accompanied by paranoia, hallucinations, and delusions. Posttraumatic stress disorder (PTSD) is a condition in which a patient experiences "flashbacks" or nightmares following exposure to a stimulus that reminds him or her of a previous negative event. Organic brain syndrome (OBS) is a general term used to describe decreased mental function secondary to a medical disease, not a psychiatric disorder. OBS is often used synonomously, but incorrectly, with dementia.

A 30-year-old man complains of difficulty breathing and a fever for the past 10 days. He is emaciated, is coughing, and has purple blotches on his trunk. This patient's clinical presentation is MOST consistent with: A: hepatitis. B: tuberculosis. C: HIV/AIDS. D: pneumonia.

*C: HIV/AIDS* Reason: Signs and symptoms of advanced HIV disease (AIDS) include weight loss, which gives the patient an emaciated appearance; persistent fever; night sweats; fatigue; and purple blotches on the skin, which are malignant lesions called Kaposi's sarcoma. The patient may have pneumonia; however, this would be the result of the immunosuppression associated with HIV/AIDS. Kaposi's sarcoma are not observed in patients with tuberculosis, pneumonia (without HIV/AIDS), or hepatitis.

You are assessing a young female and are trying to determine if she experienced a seizure or a syncopal episode. Which of the following findings is MOST consistent with a seizure? A: The episode occurred while she was standing. B: The cardiac monitor displays sinus bradycardia. C: Her heart rate is 120 beats/min. D: A bottle of lorazepam is found in her purse.

*C: Her heart rate is 120 beats/min* Reason: It can be difficult to determine whether a patient experienced a seizure or a syncopal episode (fainting). Therefore, it is important to ascertain what happened before and after the episode. Tachycardia is a common finding following a seizure, when the patient is postictal; it is caused by a sympathetic nervous system discharge that occurred during the seizure. Bradycardia often causes syncope secondary to an increase in vagal tone. Seizures occur regardless of the patient's position, whereas syncope typically occurs while the patient is standing. Most patients with a history of seizures take medications that prevent seizures, such as phenytoin (Dilantin), valproic acid (Depakote), and carbamazepine (Tegretol), among others. Although benzodiazepines (ie, diazepam [Valium], lorazepam [Ativan]) are the most common class of drugs used to terminate seizures, they are more commonly prescribed for anxiety; they are rarely prescribed to prevent seizures.

Which of the following describes the typical sequence of events that precedes cardiac arrest in a drowning episode? A: Dysrhythmias, laryngospasm, hypoxia B: Laryngospasm, dysrhythmias, hypoxia C: Laryngospasm, hypoxia, dysrhythmias D: Hypoxia, laryngospasm, dysrhythmias

*C: Laryngospasm, hypoxia, dysrhythmias* Reason: Typically, when a swimmer becomes panicked, he or she starts swallowing large amounts of water. Even a small volume of aspirated water (fresh or salt) can cause laryngospasm, which leads to hypoxia and unconsciousness. As the hypoxia worsens, cardiac dysrhythmias can develop, which leads to cardiac arrest.

Which of the following drug overdoses could be reversed with the administration of naloxone? A: Adderall B: Phenobarbitol C: Meperidine D: Midazolam

*C: Meperidine* Reason: Meperidine hydrochloride (Demerol) is a potent narcotic analgesic; its CNS depressant effects can be reversed with the administration of naloxone. Naloxone (Narcan) is a narcotic antagonist that binds to opiate/opioid receptor sites in the body, thus blocking the CNS depressant effects (ie, respiratory depression, hypotension, bradycardia) that narcotics cause. It is important to note that the effects of many narcotic/opiate drugs outlive the effects of a single dose of naloxone; therefore, repeat doses of naloxone are often needed. Midazolam (Versed) is a benzodiazepine sedative-hypnotic and anticonvulsant; it is not a narcotic. Phenobarbitol (Luminal) is a barbiturate sedative-hypnotic and anticonvulsant; it is not a narcotic. Adderall (amphetamine/dextroamphetamine) is a CNS stimulant used to treat ADHD; it is not a narcotic.

Which of the following ECG abnormalities is associated with hypothermia? A: Flat T waves B: Peaked T waves C: Osborn wave D: Sine wave

*C: Osborn wave* Reason: If a patient's core body temperature falls below 90°F (32.2°C), an Osborn wave (J wave) may be observed on the ECG. An Osborn wave is a positive deflection that causes elevation of the J point (intersection of the QRS complex and ST segment) above the isoelectric line. A sine wave (sinusoidal waveform), which may be observed on the ECG in patients with severe hyperkalemia, is characterized by a widening QRS that merges with its corresponding T wave. Tall, peaked T waves are also an ECG indicator of significant hyperkalemia. Flattened or inverted T waves, a prominent U wave, and a wide PR interval, are ECG indicators of significant hypokalemia.

What portion of the brain regulates a person's level of consciousness? A: Medulla oblongata B: Cerebrum C: Reticular activating system D: Cerebellum

*C: Reticular activating system* Reason: The reticular activating system (RAS), which is located within the brainstem, controls a person's state of awareness and level of consciousness. Coma following severe head injury indicates brainstem involvement and injury to the RAS. The pons and medulla, also located within the brainstem, are the respiratory centers. The cerebrum, the largest part of the brain, controls thought processes and memory. The cerebellum, located inferoposterior to the cerebrum, regulates coordination, balance, and equilibrium.

When assessing a patient with a preexisting mental illness, which of the following observations would be the MOST suggestive of the potential for violence? A: Sitting, crying, unable to recall birthday B: Standing facing you, arms crossed C: Sitting, clenched fists, erratic speech D: Large body size, sitting, flat affect

*C: Sitting, clenched fists, erratic speech* Reason: The potential for violence exists on every call the paramedic responds to. People respond to stress differently; anyone involved in an incident—the patient, family members, or bystanders—can become angry or violent. In patients with certain preexisting mental illnesses (eg, intermittent explosive disorder, schizophrenia), the potential for violence is clearly higher, and the paramedic must be able to recognize certain behaviors that indicate this. A "fighting stance" position—the patient is not face-to-face with you, he or she is standing sideways—is an engaging position, indicating that the patient could lunge at you at any moment. Other warning signs include a tense appearance; clenched fists; pacing; agitation; and erratic, loud, or obscene speech. Crying and a flat affect are indicative of depression; however, this does not mean that the depressed patient cannot suddenly turn violent. There is absolutely no correlation between a patient's physical size and his or her potential for violence.

Your assessment of a patient reveals a diffuse petechial rash. Which of the following hematologic disorders does this indicate? A: Hemolytic anemia B: Polycythemia vera C: Thrombocytopenia D: Leukopenia

*C: Thrombocytopenia* Reason: Of the conditions listed, only one would cause a petichial rash. Thrombocytopenia, a reduction in the number of circulating platelets, can cause cutaneous bleeding and bleeding from the mucous membranes (ie, nosebleeds, rectal bleeding). Petechiae, tiny purple or red spots that appear on the skin, is caused by bleeding within the skin or under the mucous membranes. Localized petichiae may be harmless; however, a diffuse petichial rash indicates significant thrombocytopenia. Leukopenia is a reduction in the number of white blood cells (leukocytes); this condition places the patient at increased risk for infection. Polycythemia vera, also called primary polycythemia, is a hematologic disorder in which the bone marrow makes too many red blood cells; it may also result in an overproduction of white blood cells and platelets. Hemolytic anemia is a form of anemia caused by hemolysis, the abnormal breakdown (lysis) of red blood cells.

A 52-year-old man has a headache and visual disturbances that have progressively worsened over the past 3 months. These symptoms are MOST consistent with: A: a subdural bleed. B: an epidural hematoma. C: a cerebral neoplasm. D: bacterial meningitis.

*C: a cerebral neoplasm* Reason: The patient's symptoms are consistent with a space-occupying intracranial lesion such as a neoplasm (tumor or growth), which typically presents with a headache, visual disturbances, and other symptoms that progressively worsen over a period of several months. In some patients, a new-onset of seizures may be the only presenting sign of a brain tumor. Subdural hemorrhages commonly present with symptoms within 12 to 24 hours following head trauma. An epidural hematoma presents with symptoms immediately following a head injury and causes rapid clinical deterioration. Patients with bacterial meningitis also experience a rapid progression of symptoms.

Angioedema in a patient with a severe allergic reaction poses an immediate threat to life secondary to: A: compartment syndrome. B: intracranial pressure. C: airway compromise. D: congestive heart failure.

*C: airway compromise.* Reason: Angioedema, caused by the release of histamines, is the result of vascular fluid leakage into the subcutaneous layers of the skin. It is prominent in the face and neck area in patients with severe allergic reactions and can pose a significant threat to the airway.

The clinical presentation of thyroid storm MOST closely resembles that of: A: heroin overdose. B: myxedema. C: amphetamine use. D: ketoacidosis.

*C: amphetamine use* Reason: Thyroid storm is a life-threatening condition that may occur in patients with hyperthyroidism. Hyperthyroidism is a condition in which the thyroid gland produces too much of the thyroid hormones triiodothyronine (T3) and thyroxine (T4). T3 and T4 play a key role in controlling the body's basal metabolic rate (BMR). When produced in excess, these thyroid hormones produce signs and symptoms of adrenergic hyperactivity (ie, tachycardia, sweating, palpitations, anxiety). In thyroid storm, these clinical signs would be more severe, including profound tachycardia, dysrhythmias, agitation and paranoia, and cardiovascular collapse. Amphetamines also cause adrenergic hyperactivity; of the choices listed, amphetamine use and thyroid storm would produce the most similar clinical signs. Heroin, a narcotic, causes central nervous system depression, resulting in a decreased level of consciousness, hypoventilation, bradycardia, and hypotension. Advanced hypothyroidism, called myxedema, results from a severe deficiency of T3 and T4. In contrast to thyroid storm, myxedema does not present with signs of adrenergic hyperactivity.

You are called to transport a patient from an urgent care clinic to the emergency department. When reviewing the lab results from the clinic, you note that the patient's thyroid-stimulating hormone (TSH) level is very high. This indicates: A: Graves disease. B: Cushing syndrome. C: an underactive thyroid. D: elevated T3/T4 levels.

*C: an underactive thyroid* Reason: Thyroid-stimulating hormone (TSH), a hormone produced by the pituitary gland, stimulates the thyroid gland to produce triiodothyronine (T3) and thyroxine (T4). If the thyroid produces too little of these key hormones, TSH levels will increase in an effort to stimulate the thyroid. Thus, an elevated TSH level indicates hypothyroidism. If the thyroid produces excess T3 and T4, as is the case with Graves disease (hyperthyroidism), TSH levels would be correspondingly low. Cushing syndrome is not a disease of the thyroid; it is caused by an excess of cortisol production by the adrenal glands or by excessive use of cortisol or similar corticosteroid (glucocorticoid) hormones, such as prednisone, hydrocortisone, or methylprednisolone.

You respond to a residence for an overdose. While en route, the dispatcher advises you that CPR is in progress. A second paramedic unit is dispatched, but their response to the scene will be delayed. When you arrive at the scene, an emergency medical responder is performing CPR on the patient, a 30-year-old male, who apparently ingested an unknown quantity of Darvon. After confirming apnea and pulselessness, you should direct resumption of CPR and then: A: locate the medication bottle to confirm what he evidently ingested. B: establish vascular access and administer 2 mg of naloxone. C: apply the ECG electrodes and assess the patient's cardiac rhythm. D: insert an advanced airway device to ensure adequate ventilation.

*C: apply the ECG electrodes and assess the patient's cardiac rhythm* Reason: After confirming cardiac arrest, resume CPR and assess the patient's cardiac rhythm. Although he ingested propoxyphene (Darvon), a narcotic, your first priority is to determine the need for defibrillation. If indicated, deliver a single shock and immediately resume CPR starting with chest compressions. Reassess the patient every 2 minutes and repeat defibrillation as needed. With your limited resources (delayed response of the backup crew), you must prioritize additional treatment. If bag-mask ventilation (with an oral airway inserted) is adequate, establish vascular access. If bag-mask ventilations are inadequate, insert an advanced airway device and then establish vascular access. After vascular access is established, give a vasopressor (epinephrine or vasopressin). Further treatment should be guided by the patient's cardiac rhythm. Naloxone (Narcan), in a dose of 2 mg, should be given early. When the backup crew arrives, assign someone to search for medication bottles. Whether your resources are few or many, it is critical to perform high-quality CPR with limited interruptions and to defibrillate every 2 minutes if indicated.

You are called to an assisted living facility for a sick resident. The patient, a 70-year-old woman, reports tinnitus and difficulty concentrating. The patient's neighbor, who is present at the scene, tells you that the patient has consumed five or six cups of ice over the past hour. You should suspect: A: acute leukemia. B: lymphoma. C: chronic anemia. D: polycythemia.

*C: chronic anemia* Reason: Tinnitus (ringing in the ears), a craving for ice, and difficulty concentrating are hallmark findings of chronic anemia. Anemia is defined as a deficiency of red blood cells. Other findings may include a headache, dizziness, and tachycardia. Polycythemia, an overproduction of red blood cells, causes flushing of the skin; it is not commonly associated with the clinical signs that this patient is exhibiting. Leukemia and lymphoma are types of cancer that commonly manifest with persistent weakness, fever, and swollen lymph nodes (lymphadenopathy).

Prehospital treatment for a black widow spider bite includes: A: 10 mL of calcium chloride. B: 2 mg/kg of 10% calcium gluconate. C: antivenin derived from horse serum. D: 2.5 to 10 mg of diazepam.

*D: 2.5 to 10 mg of diazepam* Reason: In addition to airway, breathing, and circulatory support, narcotics and muscle relaxants/sedatives are the prehospital mainstays of therapy when treating a black widow spider bite. To treat the muscle spasms, give 2.5 to 10 mg of diazepam (Valium); lorazepam (Ativan) may also be used. Some EMS system protocols may call for 0.1 to 0.2 mg/kg of calcium gluconate. Narcotics, such as fentanyl or morphine, can be used for pain relief. Calcium chloride is not effective in the treatment of a black widow spider bite. An antivenin is available for treatment of black widow spider bites; however, this is not a prehospital intervention. Furthermore, this antivenin is typically reserved for the young and old who have severe envenomation; it also has a high incidence of allergic reactions inherent in all equine-derived (horse serum) antivenin.

A middle-aged female with a history of hypertension presents with an acute onset of tearing abdominal pain. She is conscious and alert, but restless. Her BP is 86/56 mm Hg, pulse rate is 120 beats/min, and respirations are 28 breaths/min. You should: A: administer 5 mg of morphine to relieve her pain. B: vigorously palpate her abdomen for a pulsating mass. C: determine if her femoral pulses are of equal strength. D: increase her blood pressure with IV fluid boluses.

*C: determine if her femoral pulses are of equal strength* Reason: Your patient may be experiencing acute dissection of the abdominal aorta. A unilateral femoral pulse deficit (one femoral pulse is weaker than the other) may be observed in patients with dissection of the abdominal aorta—especially if the dissection occurs where the common iliac arteries branch. Vigorous palpation is not indicated for anyone with abdominal pain; it only causes further pain and anxiety, and could aggravate the patient's condition—especially if an aortic dissection is present. The goal of IV fluid therapy is to maintain adequate perfusion; this is most reliably assessed by noting the patient's mental status and quality of peripheral pulses. The blood pressure needed to maintain adequate perfusion varies from person to person. Establish at least one large-bore IV and be prepared to give fluid boluses if needed (ie, mental status decreases, BP falls profoundly, peripheral pulses markedly weaken). Narcotic analgesia (ie, morphine, fentanyl) may be given to patients with severe abdominal pain; however, this patient is hypotensive, so narcotic analgesia is contraindicated.

A 59-year-old female complains of a headache. She is conscious and alert, has a patent airway, and is breathing adequately. You should: A: assess her pupils for equality and reactivity. B: inquire about a history of stroke. C: determine when her headache began. D: ask her if she takes any medications.

*C: determine when her headache began* Reason: After performing your primary assessment and treatment of a responsive medical patient, you should perform a secondary assessment. This begins by assessing the history of the present illness, which is an elaboration of the chief complaint. The OPQRST mnemonic is a helpful template to use for this purpose. Ask her when her headache began—that is, when was its onset? Did it begin acutely, or has it progressively been getting worse? An acute onset of a headache suggests a more serious problem than one that has gradually progressed. Next, determine if anything provokes or palliates the headache, ask the patient to describe the pain (eg, dull or sharp), assess the severity of her pain (0-10 pain scale), and determine the time frame of the pain. Further history information can be obtained using the SAMPLE mnemonic, during which you can inquire about her past medical history (stroke, migraines). The secondary assessment is driven by information gathered during the primary assessment and history-taking phase and should focus on the patient's chief complaint. For the patient complaining of a headache, this would be an appropriate time to assess pupils, gross neurologic functions, and so on. A systematic patient assessment will facilitate the provision of the most appropriate treatment. After all, the treatment you provide is only as good as the assessment you perform.

Prehospital treatment for a patient with a blood glucose reading of 400 mg/dL and polyuria includes: A: 10 units of insulin via the subcutaneous route. B: 40 mg of furosemide via the intravenous route. C: fluid rehydration with an isotonic crystalloid. D: 25 g of 50% dextrose via the intravenous route.

*C: fluid rehydration with an isotonic crystalloid* Reason: Patients with blood glucose levels of 400 mg/dL and polyuria are dangerously close to hyperglycemic ketoacidosis, if not already there. High levels of blood glucose promote an osmotic diuresis, which explains the excessive urination (polyuria); this results in significant dehydration. Prehospital treatment is aimed at rehydrating the patient with an isotonic crystalloid solution (ie, normal saline). The patient definitely needs insulin; however, it is rarely given in the prehospital setting. Clearly, additional glucose is not indicated for a patient with documented hyperglycemia. Furosemide (Lasix), a loop diuretic, is contraindicated in patients with dehydration or hypovolemia.

You are called to a local park where a 30-year-old man has collapsed. When you arrive, you find that the patient is disoriented. His skin is hot and moist and his respirations are rapid and shallow. You should suspect: A: heat cramps. B: heat exhaustion. C: heatstroke. D: heat prostration.

*C: heatstroke* Reason: Hot (dry or moist) skin, tachypnea, and an altered mental status are indicative of heatstroke. Two major factors help differentiate heat exhaustion from heatstroke. In heat exhaustion, sometimes called heat prostration, the patient is usually conscious and alert with cool, moist skin. In heatstroke, the patient's mental status is typically altered and his or her skin is hot to the touch. Although the patient in heatstroke is not actively sweating, he or she may have residual perspiration on the skin; this is especially true in patients with extertional heatstroke. Do not rule out heatstroke just because the patient's skin is moist.

A 71-year-old female presents with an altered mental status. According to her husband, she has type II diabetes and hypertension, and was recently diagnosed with a urinary tract infection. Her medications include Glucophage, Zestril, and Bactrim. The patient is confused, and has slurred speech and poor skin turgor. Her blood pressure is 112/60 mm Hg, pulse is 120 beats/min and weak, and respirations are 30 breaths/min and shallow. Her blood glucose level reads "high," but you do not notice any unusual odors on her breath. The primary cause of this patient's problem is MOST likely: A: primary adrenal insufficiency. B: hyperglycemic ketoacidosis. C: hyperosmolar hyperglycemic nonketosis. D: severe dehydration due to an infection.

*C: hyperosmolar hyperglycemic nonketosis* Reason: Hyperosmolar hyperglycemic nonketotic coma (HHNC), most often seen in patients with type II diabetes, is marked by hyperglycemia, hyperosmolarity, and an absence of significant ketosis. Despite its name, coma occurs in fewer than 10% of cases. Most patients present with severe dehydration and focal or global neurologic deficits. HHNC often develops in patients who have a secondary illness, usually an infection, that leads to reduced fluid intake. Hyperglycemia and hyperosmolarity lead to osmotic diuresis, a fluid shift to the intravascular space, and intracellular dehydration. Unlike diabetic ketoacidosis (DKA), HHNC does not cause ketoacidosis; this explains the absence of unusual breath odors (ie, acetone) in this patient. Factors that may explain absent ketoacidosis include limited ketogenesis (ketone production due to fatty acid breakdown), low free fatty acid levels, and the availability of insulin in amounts sufficient to inhibit ketogenesis but insufficient to prevent hyperglycemia. Blood glucose levels greater than 600 mg/dL are common with HHNC. The patient's presentation is not consistent with primary adrenal insufficiency (Addison's disease), symptoms of which includes weight loss, vomiting, diarrhea, muscle and joint pain, abdominal pain, and increased skin pigmentation.

A patient with a blood glucose level of 650 mg/dL would be expected to present with: A: hypercarbia, anorexia, hyperactivity, and diaphoresis. B: tachypnea, anuria, alkalosis, and a bounding pulse. C: hyperpnea, dehydration, warm skin, and tachycardia. D: hypopnea, oliguria, abdominal pain, and vomiting.

*C: hyperpnea, dehydration, warm skin, and tachycardia* Reason: Patients with diabetic coma (diabetic ketoacidosis [DKA], hyperglycemic crisis) have a significantly elevated blood glucose level. In the absence of insulin, glucose cannot enter the cell; this results in metabolic acidosis secondary to ketoacid production from cellular fat metabolism. Excess blood glucose levels promote an osmotic diuresis, resulting in excessive urination (polyuria) and significant dehydration. Signs of dehydration include warm, dry skin with poor turgor; and a rapid, weak pulse. The respiratory buffer system attempts to eliminate ketones from the blood by increasing the rate and depth of respirations (tachypnea and hyperpnea); this is called Kussmaul respirations, and is associated with an acetone odor on the patient's breath. Patients with DKA typically have a decreasing level of consciousness that progresses to coma; hyperactivity would not be observed.

Disequilibrium syndrome, a condition associated with dialysis, manifests with signs and symptoms of: A: severe hypokalemia. B: excessive catecholamine release. C: increased intracranial pressure. D: high serum potassium levels.

*C: increased intracranial pressure* Reason: During dialysis, the concentration of urea in the blood is lowered rapidly, while the solute concentration of the cerebrospinal fluid (CSF) remains high. Water, of course, moves by osmosis from a solution of lower concentration into a solution of higher concentration. Thus, as a consequence of dialysis, water initially shifts from the bloodstream into the CSF, thereby increasing intracranial pressure (ICP). In such a circumstance, the patient may experience typical signs and symptoms of mildly increased ICP, including nausea, vomiting, headache, and confusion. After a few hours, the fluid will re-equilibrate between the bloodstream and CSF, and the patient's symptoms will spontaneously resolve. In the field, however, it may be impossible to distinguish between disequilibrium syndrome and subdural hematoma, to which dialysis patients are particularly prone. Signs and symptoms of hyperkalemia (high serum potassium levels) are commonly encountered in patients with chronic renal failure, especially if the patient's potassium intake is increased or if a dialysis treatment is missed. Hyperkalemic patients commonly present with profound muscular weakness; on the ECG, peaked T waves and prolonged QRS complexes are commonly observed. Hypokalemia (low serum potassium level) may occur as a consequence of overaggressive dialysis, and would most likely be encountered during or immediately following a dialysis cycle. Hypokalemic patients are often hypotensive and commonly present with cardiac dysrhythmias (usually bradycardias). Tachycardia and hypertension—signs of excessive catecholamine release—are not commonly associated with dialysis.

A 40-year-old female presents with diffuse abdominal cramping, nausea, vomiting, and diarrhea. She tells you that this began the day before, and is worse today. She is conscious and alert, lightheaded, and has a resting heart rate of 120 beats/min. Her temperature is 97.7°F orally, blood pressure is 110/70 mm Hg, and oxygen saturation is 95% on room air. She denies any past medical history, but states that she has been taking ibuprofen for a headache. The MOST appropriate treatment for this patient includes: A: high-flow oxygen via nonrebreathing mask, an IV of normal saline, sequential 20 mL/kg fluid boluses, and 2 mg of morphine. B: obtaining orthostatic vital signs, oxygen as tolerated, establishing vascular access, and setting the flow rate at 125 mL/hr. C: oxygen as tolerated via nasal cannula, an IV of normal saline, a 500 mL fluid bolus, and 12.5 mg of promethazine IV. D: supplemental oxygen, an IV of normal saline set to keep the vein open, and 25 to 50 mg of diphenhydramine IV.

*C: oxygen as tolerated via nasal cannula, an IV of normal saline, a 500 mL fluid bolus, and 12.5 mg of promethazine IV* Reason: The patient likely has gastroenteritis, inflammation or infection of the GI tract, and is dehydrated due to vomiting and diarrhea. An absence of fever suggests a viral cause, but may be the result of the antipyretic effects of the ibuprofen she has been taking. The norovirus (Norwalk virus) causes most cases of viral gastroenteritis in adults. Treatment includes fluid rehydration, oxygen as needed, and transport. Establish IV access, give a 500 mL fluid bolus, and reassess the patient. Give additional fluid boluses as needed. The patient is not in shock; therefore, sequential 20 mL/kg fluid boluses are not indicated. Nausea and vomiting is treated with an antiemetic such as promethazine (Phenergan), 12.5 to 25 mg, or ondansetron (Zofran), 4 mg. Oxygen via nasal cannula is appropriate for this patient; she is conscious and alert and has a room air Sp02 of 95%. Avoid putting an oxygen mask on nauseated patients unless significant hypoxemia is present. Lightheadedness and resting tachycardia are obvious signs of dehydration; orthostatic vital signs will yield little additional information. Analgesia is usually not necessary for patients with gastroenteritis. Furthermore, morphine often causes nausea and vomiting, an undesirable side effect in an already dehydrated patient. Diphenhydramine (Benadryl), an antihistamine, is used to treat allergic reactions, although it does possess antiemetic properties.

You receive a call for a 55-year-old female who is ill. The patient's husband tells you that she is an alcoholic, and has been "on the wagon" for about a week. The patient is conscious, but confused and restless. Her skin is hot to the touch, she is profusely diaphoretic, and her hands are shaking. Her blood pressure is 180/90 mm Hg, pulse is 120 beats/min, respirations are 24 breaths/min, and oxygen saturation is 98% on room air. You should: A: provide reassurance, establish vascular access and give a saline bolus, give oxygen via nonrebreathing mask, administer 5 mg of diazepam to prevent seizures, and transport. B: establish vascular access, administer 100 mg of thiamine and 25 g of 50% dextrose IV push, apply oxygen via nasal cannula, orient her and provide reassurance, and transport. C: provide emotional support, administer oxygen via nasal cannula, establish vascular access, monitor her cardiac rhythm, assess her blood glucose level, and transport. D: administer high-flow oxygen via nonrebreathing mask, establish vascular access, give 2 to 5 mg of midazolam for sedation, monitor her cardiac rhythm, and transport.

*C: provide emotional support, administer oxygen via nasal cannula, establish vascular access, monitor her cardiac rhythm, assess her blood glucose level, and transport* Reason: A potentially fatal complication of alcohol withdrawal is delirium tremens (DTs). Symptoms usually start 48 to 72 hours after the last alcohol intake (longer in some cases). Mortality rates as high as 15% have been reported. Signs include confusion, tremors, restlessness, fever, diaphoresis, frightening hallucinations, autonomic hyperactivity (eg, hypertension and tachycardia), and in some cases, seizures. If the patient is severely dehydrated, hypotension may be present. Treatment is aimed at protecting the patient from injury and supporting the ABCs. Hallucinations associated with DTs often make for an agitated or combative patient; try to keep the patient calm and oriented, and provide emotional support and reassurance. Give oxygen (a nasal cannula is appropriate in this case), establish IV access (a BP of 180/90 mm Hg does not warrant fluid boluses), and transport. Assess the blood glucose level of any patient with an altered mental status and administer IV dextrose if hypoglycemia is present. Some patients with DTs may develop cardiac dysrhythmias, so ECG monitoring is essential. Benzodiazepines (eg, Valium, Ativan) are not used to prevent seizures; they are used to terminate seizures if they occur. Sedation of a confused patient is not advisable; it may mask other neurological signs.

You would expect to encounter trismus in a patient with: A: mumps. B: meningitis. C: tetanus. D: rabies.

*C: tetanus* Reason: The most common presenting manifestation in patients with tetanus is trismus—spasm of the jaw muscles that causes difficulty opening the mouth—hence the term "lockjaw." Tetanus is a potentially fatal disease of the central nervous system caused by the bacterium Clostridium tetani. If spores of C tetani enter the circulatory system, they multiply and produce a neurotoxin called tetanospasmin, which attacks the nerves that regulate muscular activity. As tetanospasmin circulates more widely, it interferes with the normal activity of nerves throughout the body, resulting in generalized muscle spasms. Any open wound that is contaminated with soil, saliva, or feces—especially if not properly cleaned—and puncture wounds from nonsterile items such as nails or needles, are at high risk of being infected with C tetani. Rabies, mumps, and meningitis—all of which can be fatal—are typically not associated with trismus.

When the body's blood glucose level falls, such as following strenuous exercise: A: the body's cells release their glucose stores into the bloodstream to maintain homeostasis. B: the cells metabolize fat, which produces a weaker form of glucose that the body can still utilize. C: the pancreas secretes glucagon, which stimulates the liver to convert glycogen to glucose. D: the pancreatic beta cells cease insulin production altogether until the blood sugar level increases.

*C: the pancreas secretes glucagon, which stimulates the liver to convert glycogen to glucose* Reason: When the body's blood glucose level falls, the alpha cells of the pancreas secrete the hormone glucagon, which stimulates the liver to convert glycogen to glucose—a process called glycogenolysis. The liver then releases glucose into the bloodstream, thereby increasing circulating glucose levels. The body's cells do not release glucose into the bloodstream; like the brain, the cells need glucose to survive. Cellular fat metabolism results in the formation of ketoacids, which occurs when insulin supplies are insufficient or absent and glucose accumulates in the blood because it is unable to enter the cell (eg, hyperglycemic ketoacidosis).

You are assessing a patient who presents with a melena and abdominal pain. Which of the following medical history findings is the MOST clinically significant? A: Type II diabetes mellitus B: Hypertension C: von Willebrand disease D: Hashimoto disease

*C: von Willebrand disease* Reason: All of the diseases listed are clinically significant. However, the presence of von Willebrand disease in a patient with internal or external bleeding should concern you the most. Von Willebrand disease, a bleeding disorder similar to hemophilia, is caused by a deficiency of von Willebrand factor (vWF), a blood clotting protein. vWF circulates attached to factor VIII, another blood clotting protein. As a consequence of von Willebrand disease, the normal process of hemostasis is interrupted and the patient will continue to bleed. Hashimoto disease, also called Hashimoto thyroiditis, is a cause of hyperthyroidism; it is an autoimmune disorder that affects the thyroid-stimulating hormone (TSH) receptors. While hypertension, Hashimoto disease, and type II diabetes can certainly complicate the clinical picture of any sick or injured patient, bleeding disorders in a patient with active bleeding would clearly be the most detrimental.

A 19-year-old female ingested an unknown quantity of Elavil and Ativan approximately 3 hours ago. She is slow to answer your questions and her speech is slurred. Her BP is 80/60 mm Hg, her pulse is 140 beats/min, and her respirations are 22 breaths/min. The ECG reveals sinus tachycardia with QRS complexes that measure 0.08 seconds. Treatment should include: A: 1 mEq/kg of sodium bicarbonate. B: 0.2 to 0.3 mg of flumazenil IV push. C: 1 to 2 g/kg of activated charcoal. D: 20 mL/kg sequential IV fluid boluses.

*D: 20 mL/kg sequential IV fluid boluses* Reason: Amitriptyline (Elavil) is a tricyclic antidepressant (TCA). In prescribed doses, TCAs are effective in treating depression; when taken in excess, they can be lethal. Common symptoms of TCA toxicity include altered mental status, dysrhythmias (usually sinus tachycardia or supraventricular tachycardia), blurred vision, and pupillary dilation (mydriasis). Severe toxicity causes respiratory depression, QRS and QT interval widening, ventricular dysrhythmias, hypotension, pulmonary edema, and seizures. Treatment includes high-flow oxygen (assist ventilations if the patient's breathing is inadequate), sequential 20 mL/kg IV fluid boluses to treat hypotension (monitor for signs of pulmonary edema), continuous cardiac monitoring, and prompt transport. Sodium bicarbonate is indicated if QRS widening occurs; contact medical control or follow your local protocols regarding the dose. Alkalinizing the blood increases protein binding and may decrease the QRS width, stabilize dysrhythmias, and increase blood pressure. Flumazenil (Romazicon), a benzodiazepine antagonist, should NOT be given to patients who have coingested TCAs and benzodiazepines (ie, lorazepam [Ativan], diazepam [Valium]); it may unmask seizures. Activated charcoal is not indicated for this patient; her symptoms clearly indicate systemic absorption of the drug and her mental status is altered.

Following ingestion of a toxic dose of acetaminophen, right upper quadrant abdominal pain typically begins within: A: 72 to 96 hours B: 12 to 24 hours C: 4 to 14 days D: 24 to 72 hours

*D: 24 to 72 hours* Reason: Acetaminophen, the active ingredient in Tylenol, can cause liver failure and death if a toxic dose is ingested. A unique aspect of acetaminophen toxicity is that its signs and symptoms appear in four distinct stages. Stage I (less than 24 hours) symptoms include nausea, vomiting, anorexia, pallor, and malaise. Stage II (24 to 72 hours) symptoms include right upper quadrant (RUQ) abdominal pain and abdominal tenderness to palpation. Stage III (72 to 96 hours) symptoms include metabolic acidosis, renal failure, coagulopathies, and recurring GI symptoms. During Stage IV (4 to 14 days [or longer]), recovery slowly begins or liver failure progresses and the patient dies. The antidote for acetaminophen toxicity is acetylcysteine (Acetadote); ideally, it should be given less than 8 hours following ingestion.

What is the MOST appropriate dose and route of diphenhydramine for a patient who is experiencing a severe allergic reaction? A: 25 to 50 mg IM B: 0.3 to 0.5 mg IM C: 0.3 to 0.5 mg SC D: 25 to 50 mg IV

*D: 25 to 50 mg IV* Reason: Diphenhydramine (Benadryl) is an antihistamine and is a second line medication to epinephrine in patients with severe allergic reactions (eg, anaphylactic shock). The dose is 25 to 50 mg IV. In patients with anaphylactic shock, intramuscular (IM) injections will not be as effective since circulation of blood through the muscles is diminished secondary to the peripheral shunting of blood. As a result, drugs given by this route would have a delayed onset of action. It would be appropriate to give diphenhydramine IM to patients with mild to moderate allergic reactions because peripheral perfusion is generally adequate.

Treatment for a patient with a severe migraine headache, nausea, and vomiting includes: A: heat packs to the forehead. B: transport in a supine position. C: 2 mg of morphine sulfate IV. D: 5 mg of prochlorperazine IV.

*D: 5 mg of prochlorperazine IV* Reason: Prochlorperazine (Compazine), a phenothiazine, is an antiemetic medication that will not only relieve nausea and vomiting from a migraine headache but has been found to be effective in terminating the headache itself. Patients with headaches such as migraines or cluster headaches generally prefer to lay flat (unless they are actively vomiting) with the lights dimmed. In some cases, chemical ice packs or a cool wash cloth to the forehead may afford the patient additional pain relief. Narcotic analgesia should generally be avoided in patients with a headache, even if the patient believes that he or she knows the cause. If the patient is experiencing an intracranial hemorrhage, vasodilator drugs (ie, morphine, fentanyl) could exacerbate their condition.

Which of the following patients is at greatest risk for hypothermia? A: 65-year-old man with coronary artery disease B: 45-year-old man with hyperglycemia C: 60-year-old woman with Cushing syndrome D: 55-year-old woman with hypothyroidism

*D: 55-year-old woman with hypothyroidism* Reason: Hypothyroidism is a condition in which the thyroid gland produces too little T3 (triiodothyronine) and T4 (thyroxine), resulting in a decrease in the metabolic rate. Any time the metabolic rate decreases, heat energy production is reduced; therefore, the patient is prone to hypothermia. Cushing syndrome is caused by excessive cortisol production by the adrenal glands or by excessive use of cortisol or other similar glucocorticoid hormones (ie, prednisone, hydrocortisone, methylprednisolone). This increase in cortisol would cause an increase in the metabolic rate. Patient's with Cushing syndrome are not at risk for hypothermia, nor are patients with hyperglycemia or coronary artery disease.

Which of the following assessment findings is MOST indicative of peritoneal irritation? A: A relief of pain when the patient moves around frequently B: Pain that increases when the patient is placed on his or her side C: Pain that is referred to the shoulder, neck, or scapulae D: A decrease in pain when drawing the knees into the abdomen

*D: A decrease in pain when drawing the knees into the abdomen* Reason: The classic position for the patient with peritoneal inflammation or irritation is recumbent with the knees drawn up into the chest (fetal position). This position takes much of the stress off of the abdominal muscles, thereby affording some pain relief. Even small bumps in the road can cause intense pain in patients with peritonitis.

A 16-year-old boy has a severe headache and vomiting that has progressively worsened over the past 36 hours. Which of the following questions would be MOST important to ask him? A: Do you have any abdominal pain or diarrhea? B: Do you have a history of hypertension? C: Is there a history of meningitis in your family? D: Have you experienced a recent head injury?

*D: Have you experienced a recent head injury?* Reason: A severe headache and vomiting that progressively worsens could indicate a subdural hemorrhage; therefore, one of the most important questions to ask the patient is if he has experienced any recent head injury, even as far back as a week. Subdural hematomas are the result of venous bleeding and can be insidious in their presentation, with symptoms that often appear hours to days after the initial injury. Hypertension is unlikely in a 16-year-old patient, and meningitis is not a hereditary disease.

A 59-year-old male with a history of hypertension and diabetes presents with dark, tarry stools. He is confused, has a blood pressure of 84/62 mm Hg, and a pulse rate of 74 beats/min and weak. Which of the following would MOST likely explain his heart rate? A: Increased parasympathetic tone in response to hypovolemia B: Inadvertent overdose of his prescribed hypoglycemic medication C: Insulin suppressing the patient's sympathetic nervous system D: A prescribed adrenergic blocking agent to treat his hypertension

*D: A prescribed adrenergic blocking agent to treat his hypertension* Reason: Adrenergic blocking medications, such as beta blockers, are commonly used to treat hypertension. Examples of beta blockers include propranolol (Inderal), metaprolol (Lopressor), atenolol (Tenormin), and labetalol (Normodyne). Beta blockers suppress the sympathetic nervous system, which reduces heart rate and myocardial contractility. The patient in this scenario is in shock due to gastrointestinal bleeding. His heart rate—which you would expect to be fast—is in a normal range. This is most likely the result of his prescribed beta blocker medication, which is blunting the sympathetic nervous system's compensatory response to shock. In hypovolemic shock, the sympathetic nervous system—not the parasympathetic nervous system—is stimulated, resulting in tachycardia, pallor, and diaphoresis, among other signs.

Which of the following assessment findings helps distinguish envenomation due to a black widow spider bite from an acute abdominal condition? A: Dizziness, nausea, vomiting, and diaphoresis B: Fever, palpable abdominal tenderness, and tachycardia C: Hematemesis, dysphagia, and significant hypotension D: Abdominal rigidity without palpable tenderness

*D: Abdominal rigidity without palpable tenderness* Reason: The venom of a black widow spider contains a neurotoxin that causes severe muscles spasms and intense pain—especially of the abdomen. Unlike an acute abdominal condition, however, the patient's abdomen is typically not tender to palpation. Dizziness, nausea, vomiting, and diaphoresis are common in patients with an acute abdominal condition and black widow spider envenomation. Fever and tachycardia in conjunction with abdominal tenderness is consistent with acute abdominal conditions such as peritonitis and appendicitis, not envenomation from a black widow spider. Hematemesis, dysphagia, and hypotension are common assessment findings in patients with ruptured esophageal varices.

An older woman presents with severe weakness, hypotension, lower back pain, and vomiting. Her husband tells you that she has not taken her prednisone in several days because she has not been feeling well. Which of the following should you suspect? A: Thyrotoxic crisis B: Pheochromocytoma C: Cushing syndrome D: Addisonian crisis

*D: Addisonian crisis* Reason: Signs and symptoms of acute adrenal insufficiency can manifest suddenly in what is called an addisonian crisis. Abrupt cessation of corticosteroid therapy (ie, prednisone, hydrocortisone) is the most common cause of an addisonian crisis. It may also be triggered by acute exacerbation of chronic adrenal insufficiency (Addison disease), usually brought on by stress, trauma, surgery, or a severe infection. In either case, cardiovascular collapse occurs due to a lack of the hormone cortisol; therefore, the chief clinical manifestation of an addisonian crisis is shock. Other signs and symptoms may include weakness; lethargy; fever; severe pain in the lower back, legs, or abdomen; and severe vomiting and diarrhea. Cushing syndrome is caused by excessive cortisol production by the adrenal cortex; it may also occur if large amounts of corticosteroids are administered. Pheochromocytoma is an adrenal tumor that causes excessive release of epinephrine and norepinephrine; patients with this condition present with hypertension and tachycardia. Thyrotoxic crisis (thyroid storm) is a condition caused by critically high thyroid hormone levels, resulting in a hypermetabolic state. Signs and symptoms include severe tachycardia, hypertension, fever, altered mental status, and possibly heart failure.

Which of the following findings is the MOST clinically significant when assessing a patient with a severe headache? A: BP of 140/88 mm Hg B: Retroorbital pressure C: Nausea or vomiting D: An unsteady gait

*D: An unsteady gait*

You are called to the residence of an elderly man whose daughter states that he is not acting right. The patient becomes combative when you attempt to assess him. He refuses supplemental oxygen and states that you are not taking him anywhere. What is the MOST appropriate course of action? A: Start an IV line and administer 25 gm of 50% dextrose. B: Gently restrain him and transport him to the hospital. C: Administer 5 mg diazepam IM to calm and sedate him. D: Calmly talk to him and try to obtain a glucose reading.

*D: Calmly talk to him and try to obtain a glucose reading* Reason: Any patient with an altered mental status should be ruled out for hypoglycemia by obtaining a blood glucose reading. Administering IV dextrose without assessing his blood glucose level first is not advisable. If he is experiencing an intracranial hemorrhage, dextrose may exacerbate his condition. Conversely, if you discover that he is hypoglycemic, this must be corrected. It is important to calmly approach any patient, regardless of his or her mental status. The assumption of a psychiatric crisis is not in the best interest of the patient. Focus on ruling out medical problems first.

A 42-year-old male presents with difficulty breathing, diffuse wheezing, urticaria, and a blood pressure of 74/44 mm Hg. His skin is cool and clammy. Given his clinical presentation, which of the following interventions would be the LEAST effective? A: 1 to 2 L saline bolus B: Diphenhydramine, 25 mg IV C: Epinephrine, 2 µg/min IV D: Epinephrine, 0.3 mg SC

*D: Epinephrine, 0.3 mg SC* Reason: The patient in this scenario is experiencing signs and symptoms of anaphylactic shock—dyspnea, wheezing, urticaria (hives). Furthermore, the fact that he is hypotensive indicates that he is in decompensated shock. Poor peripheral perfusion is evidenced by his cool, clammy skin; blood has been shunted away from the skin to the vital organs. Compared to the intravenous (IV) and intraosseous (IO) routes, medications given via the subcutaneous (SC) and intramuscular (IM) routes take longer to absorb into the central circulation—even in patients with adequate peripheral perfusion. Therefore, it would clearly stand to reason that a medication administered via the SC route to a patient with poor peripheral perfusion would have an even more delayed effect. For patients with anaphylactic shock, epinephrine should be administered via the IV or IO route. The appropriate dose is 0.1 mg (1 mL) of a 1:10,000 solution. An IV infusion of epinephrine at 1 to 4 µg/min also would be appropriate, and may avoid the need to give repeated epinephrine boluses for severely hypotensive patients. If hypotension does not respond rapidly to epinephrine, crystalloid fluid boluses should be given; as much as 1 to 2 L may be needed initially. Diphenhydramine (Benadryl), 25 to 50 mg IV, is given after epinephrine; it blocks the release of histamines that are causing the allergic reaction. Patients with mild or moderate allergic reactions who have adequate peripheral perfusion are commonly given epinephrine 1:1,000 via the SC or IM route in a dose of 0.3 to 0.5 mg (0.3 to 0.5 mL). Benadryl can also be given via the IM route.

A 16-year-old boy has a severe headache and vomiting that has progressively worsened over the past 36 hours. Which of the following questions would be MOST important to ask him? A: Do you have any abdominal pain or diarrhea? B: Is there a history of meningitis in your family? C: Do you have a history of hypertension? D: Have you experienced a recent head injury?

*D: Have you experienced a recent head injury?* Reason: A severe headache and vomiting that progressively worsens could indicate a subdural hemorrhage; therefore, one of the most important questions to ask the patient is if he has experienced any recent head injury, even as far back as a week. Subdural hematomas are the result of venous bleeding and can be insidious in their presentation, with symptoms that often appear hours to days after the initial injury. Hypertension is unlikely in a 16-year-old patient, and meningitis is not a hereditary disease.

A patient with diabetic ketoacidosis would typically present with which of the following signs and/or symptoms? A: Hypoglycemia and dehydration B: Hyperglycemia and oliguria C: Hypoglycemia and polyuria D: Hyperglycemia and dehydration

*D: Hyperglycemia and dehydration* Reason: Diabetic ketoacidosis (DKA), also referred to as diabetic coma or hyperglycemic crisis, is characterized by hyperglycemia, polyuria (excessive urination), polydipsia (excessive thirst), and polyphagia (excessive hunger). Other findings include warm, dry skin, dehydration, and deep, rapid respirations (Kussmaul respirations). The progression to DKA is typically slow, often over several hours to a few days. By contrast, insulin shock (hypoglycemic crisis) is characterized by a rapid onset, often within a few minutes.

Which of the following findings is MOST suggestive of myxedema? A: Hyperactivity B: Tachycardia C: Weight loss D: Hypothermia

*D: Hypothermia* Reason: Advanced hypothyroidism is sometimes called myxedema. Frequently, patients have localized accumulations of mucinous material in the skin, which gives the disease its name (the prefix myx- refers to "mucin," and edema means "swelling"). Myxedema manifests as a general slowing of the body's metabolic processes due to a significant reduction or absence of the thyroid hormones T3 (triiodothyronine) and T4 (thyroxine). Since the thyroid gland regulates the metabolic rate and metabolism produces heat energy, patients with myxedema are prone to hypothermia. This also explains why patients with hypothyroidism are poorly tolerant of cold temperatures. Other signs and symptoms of myxedema include lethargy, depression, bradycardia, and weight gain. In severe cases, coma and death can occur.

Which of the following statements regarding lightning injuries is correct? A: Muscle that is injured by lightning produces methemoglobin, which can damage the kidneys. B: Asystole following a lightning strike is generally intractable and responds poorly to CPR. C: Lightning is alternating current that causes severe damage to the nerves and microvasculature. D: Lightning injuries tend to resemble blast injuries more than they do high-voltage injuries.

*D: Lightning injuries tend to resemble blast injuries more than they do high-voltage injuries* Reason: Lighting carries enormous electrical current; its energy can reach 100 million volts, and peak currents can be in the range of 200,000 amps. Unlike other high-voltage electrical current, lightning is direct—not alternating—current, and the duration is measured in milliseconds. Thus, lightning injuries tend to resemble blast injuries more than they do high-voltage injuries, with damage to the tympanic membranes and air-containing internal organs being common. Lightning delivers a massive direct-current shock that depolarizes the entire heart, usually resulting in asystole. However, the heart may resume beating spontaneously shortly after the shock or after a few minutes of high-quality CPR. It is for this reason that pulseless and apneic patients who have been struck by lightning should be treated first (reverse triage); victims who are conscious following a lightning strike, although injured, are less likely to develop cardiac arrest. Muscle damage may occur following a lightning strike, and the release of myoglobin from injured muscle can damage the kidneys. Methemoglobin—an abnormal form of hemoglobin—occurs when hemoglobin is oxidized during blood decomposition or by the action of various toxic agents (ie, nitrates). Methemoglobin contains iron in the ferric state and is unable to carry oxygen to the body's tissues.

Which of the following medications is classified as a tricyclic antidepressant? A: Midazolam hydrochloride B: Buspirone hydrochloride C: Fluoxetine hydrochloride D: Nortriptyline hydrochloride

*D: Nortriptyline hydrochloride* Reason: Nortriptyline (Pamelor), amyltriptyline (Elavil), and clomipramine hydrochloride (Anafranil) are commonly prescribed tricyclic antidepressant (TCA) medications. Fluoxetine hydrochloride (Prozac) is a selective serotonin reuptake inhibitor (SSRI) that is also used to treat depression as well as obsessive-compulsive disorder. Midazolam hydrochloride (Versed) is a benzodiazepine sedative-hypnotic. Buspirone hydrochloride (Buspar) is an anxiolytic medication

At 2:30 AM, you respond to a crowded homeless shelter for a 52-year-old male who is sick. The patient complains of intense itching to his hands and axillae. Assessment of these areas reveals the presence of a rash. The patient denies any medical problems, but states that he was stung by a hornet two days ago. Which of the following should you suspect? A: Allergic reaction B: Lice C: Herpes simplex D: Scabies

*D: Scabies* Reason: This patient's presentation is classic for scabies. Scabies are caused by infection with Sarcoptes scabiei, a parasite. Infection with scabies commonly affects families, children, sexual partners, and persons in communal living (ie, homeless shelters). Signs and symptoms of scabies include nocturnal itching and the presence of a rash involving the hands, flexor aspects of the wrists, axillary folds, ankles, toes, and genital area. Lice also present with itching and irritation; however, unlike the nocturnal presentation of scabies, the symptoms of lice occur at any time of the day or night. Herpes simplex is characterized by small vesicles; a rash is not common. It is highly unlikely that the patient is experiencing an allergic reaction; he was stung by a hornet two days ago. An allergic reaction would have presented shortly following exposure—not two days later.

A 39-year-old man reports nausea, lack of coordination, and frequently recurring headaches that have been getting progressively worse over the past 2 months. He denies any past medical history or head injury. On the basis of this presentation, which of the following should you suspect? A: Ruptured cerebral arterial aneurysm B: Chronic epidural hematoma C: Acute subarachnoid hemorrhage D: Space-occupying intracranial lesion

*D: Space-occupying intracranial lesion* Reason: The worsening and persistent symptomatology that this patient is experiencing suggests a space-occupying intracranial lesion (ie, a tumor). Ruptured cerebral aneurysms and acute epidural bleeds present with a sudden onset of symptoms. Because epidural hematomas are caused by an arterial hemorrhage, they are acute, not chronic. Subdural hematomas are caused by a venous hemorrhage; therefore, they can be acute or chronic.

Prehospital treatment for cutaneous exposure to hydrofluoric acid includes: A: baking powder covered with a dry, sterile dressing. B: 20 mL of a 10% solution of calcium gluconate IV. C: 1 to 2 g of magnesium sulfate via slow IV push. D: antacid preparations, such as Mylanta or Maalox.

*D: antacid preparations, such as Mylanta or Maalox* Reason: Hydrofluoric acid is one of the strongest inorganic acids. It is mainly used for industrial purposes, such as glass etching and metal cleaning; it may also be found in home rust removers. Tissue damage following exposure to hydrofluoric acid occurs via two mechanisms: corrosive burns from the free hydrogen ions and chemical burns from tissue penetration of the fluoride ions. Fluoride ions penetrate the skin and form insoluble salts with calcium and magnesium. Soluble salts are also formed with other cations; however, they rapidly dissociate. As a result, fluoride ions release and further tissue damage occurs. Significant exposure can result in hypocalcemia, hypomagnesemia, and hyperkalemia. After ensuring your own safety and properly decontaminating the patient by irrigating the affected area with water, cutaneous (skin) exposure to hydrofluoric acid is neutralized with one of two agents: calcium gluconate gel (usually not available in the prehospital setting) or a magnesium hydroxide antacid preparation (ie, milk of magnesia, Mylanta, Maalox). Do not attempt to neutralize an acid burn with an alkaline agent (ie, baking soda/powder); this may cause further tissue damage. If the patient is hypocalcemic—as quantified by lab analysis—emergency department treatment may include 10 mL of a 10% calcium gluconate solution IV. In the prehospital setting, significant cutaneous exposure may be treated with 5 mL of 10% calcium chloride mixed in 500 mL of normal saline for IV infusion. Follow your local protocols.

A 23-year-old man was working near a wood pile when he experienced a sudden, sharp pain in his leg. Your assessment reveals that his level of consciousness is decreased and he is experiencing intense abdominal spasms. This clinical presentation is MOST consistent with the bite of a: A: coral snake. B: brown recluse spider. C: rattlesnake or other pit viper. D: black widow spider.

*D: black widow spider* Reason: This is a classic case of a black widow spider bite. The black widow spider typically can be found near wood piles or wood sheds. The patient will usually experience immediate sharp pain at the time of the bite, and then within a short period of time painful muscle spasms will develop in all of the major muscle groups, especially the abdomen. The black widow spider carries a neurotoxin in its venom, which explains the muscle spasms. If left untreated, CNS depression will continue and the patient will experience cardiovascular and respiratory system collapse. In contrast to a black widow spider bite, the bite of a brown recluse spider is usually painless, and the patient does not even realize he or she has been bitten until a red area with a small blister in the center of it appears several hours to a day later. Unlike the black widow spider, the brown recluse spider carries a cytotoxin (necrotoxin) in its venom. Cytotoxins cause tissue and cellular necrosis.

You are treating a 50-year-old male who ingested a significant amount of his prescribed propranolol. He is unresponsive, bradycardic, hypotensive, and has poor respiratory effort. In addition to assisting his ventilations, applying a cardiac monitor, and establishing vascular access, the MOST appropriate treatment for him includes: A: 2 to 3 liters of normal saline to increase his BP. B: atropine, calcium gluconate, and vasopressin. C: calcium chloride, isoproterenol, and dopamine. D: cardiac pacing, glucagon, and a vasopressor.

*D: cardiac pacing, glucagon, and a vasopressor* Reason: Propranolol (Inderal) is a beta-blocker. Toxicity is marked by profound bradycardia and hypotension. Untreated, death occurs due to cardiovascular collapse. Some patients with beta-blocker toxicity require ventilatory support. Transcutaneous cardiac pacing (TCP), though not always successful, should be initiated without delay as you are establishing vascular access. Glucagon is given to patients with beta-blocker toxicity; its positive inotropic and chronotropic effects have been shown to improve cardiac output. Vasopressors, such as epinephrine, are also used to treat beta-blocker toxicity, although higher-than-usual doses may be needed. There is currently no recommendation for the use of vasopressin for beta-blocker overdose. Your patient's bradycardia is caused by sympathetic nervous system blockade, not increased parasympathetic tone; therefore, atropine will likely be ineffective. Isoproterenol (Isuprel) is contraindicated in bradycardia, regardless of the cause; its potent vasodilator effects may exacerbate hypotension. Hypotension and bradycardia are not consistent with hypovolemia. If you give fluid boluses, use extreme caution! Too much fluid in a bradycardic patient may cause pulmonary edema. Calcium chloride is used to treat hypocalcemia and calcium channel-blocker overdose. Calcium gluconate is used to treat hypocalcemia and magnesium sulfate toxicity; the gel form of calcium gluconate is used to treat patients with hydroflouric acid burns.

Diarrhea, marked bradycardia, miosis, and hypersalivation are MOST consistent with a/an _____________ toxidrome. A: amphetamine B: sympatholytic C: opiate D: cholinergic

*D: cholinergic* Reason: The syndrome-like signs and symptoms of a poisonous agent are called a toxidrome. Toxidromes are useful for remembering the assessment and management of different substances that fall under the same clinical umbrella. Cholinergic agents (ie, Diazinon, orthene, sarin, tabun) stimulate the parasympathetic nervous system by deactivating acetylcholinesterase—an enzyme that regulates the degradation of acetylcholine. The signs and symptoms of cholinergic toxicity—that is, the toxidrome—can be remembered with the mnemonic DUMBELS, which stands for defecation, urination, miosis (pupillary constriction), bradycardia and bronchorrhea, emesis, lacrimation, and salivation. Opiate (narcotic) toxicity presents with bradycardia, respiratory depression, hypotension, and pupillary constriction. Sympatholytic (ie, alpha or beta blocker) toxicity presents with bradycardia, hypotension, and hypoglycemia, among other signs. Amphetamine (upper) toxicity presents with restlessness, pupillary dilation, tachycardia, hypertension, tachypnea, and insomnia, among other signs.

You are dispatched to a residence for a 65-year-old female whose husband reports a change in her behavior over the past several days. He states that initially she seemed forgetful, but today she is confused. Your assessment reveals an obese patient who has slow respirations; cold, dry skin; a heart rate of 50 beats/min; and a blood pressure of 90/50 mm Hg. The patient's husband tells you that his wife does not have any medical problems that he is aware of, but remarks about her weight gain over the past few months. This patient's history and clinical presentation are MOST likely the result of: A: acute dementia with hypothermia. B: exacerbation of undiagnosed hyperthyroidism. C: excess adrenal gland production of cortisol. D: decreased thyroid hormone production.

*D: decreased thyroid hormone production* Reason: Myxedema coma is a potentially life-threatening complication of hypothyroidism. Hypothyroidism is the result of decreased thyroid hormone secretion by the thyroid gland. Thyroid hormones are critical for cell metabolism and organ function. Adult hypothyroidism, often called myxedema, causes slowing of the body's metabolic processes. Symptoms are due to a slow metabolic rate (fatigue, feeling cold, weight gain, dry skin, sleepiness). Myxedema coma is an extreme manifestation of hypothyroidism marked by bradycardia, hypotension, and a decreased mental status. Family members may overlook subtle changes, but obvious changes (eg, confusion, psychosis, coma) will elicit a call to 9-1-1. Myxedema coma usually occurs during the winter in women older than 60 years of age. Hypothermia is common; fever may be absent during an infection. Hyperthyroidism results from excessive thyroid hormone production; it manifests with signs of increased metabolism (eg, tachycardia; weight loss; hot, flushed skin). Dementia is not an acute event; it is a gradual deterioration of cognitive function (eg, Alzheimer's disease). Excess cortisol secretion by the adrenal glands causes Cushing's syndrome. Signs and symptoms include weakness, fatigue, depression, hypoglycemia, darkened skin (acanthosis) on the neck, and a moon-face appearance.

Prehospital treatment for a patient with a blood glucose reading of 400 mg/dL and polyuria includes: A: 40 mg of furosemide via the intravenous route. B: 10 units of insulin via the subcutaneous route. C: 25 g of 50% dextrose via the intravenous route. D: fluid rehydration with an isotonic crystalloid.

*D: fluid rehydration with an isotonic crystalloid* Reason: Patients with blood glucose levels of 400 mg/dL and polyuria are dangerously close to hyperglycemic ketoacidosis, if not already there. High levels of blood glucose promote an osmotic diuresis, which explains the excessive urination (polyuria); this results in significant dehydration. Prehospital treatment is aimed at rehydrating the patient with an isotonic crystalloid solution (ie, normal saline). The patient definitely needs insulin; however, it is rarely given in the prehospital setting. Clearly, additional glucose is not indicated for a patient with documented hyperglycemia. Furosemide (Lasix), a loop diuretic, is contraindicated in patients with dehydration or hypovolemia.

Prehospital treatment for a patient in ventricular fibrillation who has a core body temperature of less than 86°F (30°C) includes: A: hyperventilation with warm humidified oxygen. B: administering lidocaine instead of amiodarone. C: doubling the dose of all medications. D: limiting defibrillation to one attempt only

*D: limiting defibrillation to one attempt only* Reason: Although severely hypothermic patients in cardiac arrest usually does not respond to conventional ACLS therapies, one shock (360 monophasic joules or equivalent biphasic) can be attempted if the patient is in V-Fib or pulseless V-Tach. If the patient does not respond to one shock, further defibrillation attempts should be deferred; the paramedic should focus on providing high-quality CPR with minimal interruptions, airway management (do NOT hyperventilate the patient), rewarming per local protocol, and prompt transport. If the patient's core body temperature is less than 86°F (30°C), cardiac medications (ie, epinephrine, amiodarone, lidocaine) will be ineffective because the patient's metabolic rate is so low. Furthermore, they can accumulate to toxic levels if given repeatedly. For these reasons, cardiac medications should be withheld until the patient is properly rewarmed at the hospital.

Prehospital treatment for a patient in ventricular fibrillation who has a core body temperature of less than 86°F (30°C) includes: A: doubling the dose of all medications. B: administering lidocaine instead of amiodarone. C: hyperventilation with warm humidified oxygen. D: limiting defibrillation to one attempt only.

*D: limiting defibrillation to one attempt only* Reason: Although severely hypothermic patients in cardiac arrest usually does not respond to conventional ACLS therapies, one shock (360 monophasic joules or equivalent biphasic) can be attempted if the patient is in V-Fib or pulseless V-Tach. If the patient does not respond to one shock, further defibrillation attempts should be deferred; the paramedic should focus on providing high-quality CPR with minimal interruptions, airway management (do NOT hyperventilate the patient), rewarming per local protocol, and prompt transport. If the patient's core body temperature is less than 86°F (30°C), cardiac medications (ie, epinephrine, amiodarone, lidocaine) will be ineffective because the patient's metabolic rate is so low. Furthermore, they can accumulate to toxic levels if given repeatedly. For these reasons, cardiac medications should be wittheld until the patient is properly rewarmed at the hospital.

A young male intentionally ingested unknown quantities of ibuprofen, aspirin, Lortab, and Anafranil. He is unresponsive, and is hypoventilating, bradycardic, and hypotensive. Which of these drugs is the MOST likely cause of his clinical presentation? A: Anafranil B: Ibuprofen C: Aspirin D: Lortab

*D: sit him forward and perform a finger stick to assess his blood glucose level* Reason: Assess the blood glucose level (BGL) of any patient with altered mentation. Don't be so quick to assume intoxication or mental illness, regardless of the history given to you by others. Hypoglycemic patients can present with mental status changes ranging from bizarre behavior to coma. Plus, your patient has other signs of hypoglycemia (eg, tachypnea, tachycardia, diaphoresis). Hypoglycemic patients with abnormal mentation are unaware of their behavior, but can still pose a safety threat. In this situation, sit the patient forward, leaving him handcuffed, and perform a finger stick so you can assess his BGL. If he is hypoglycemic, establish vascular access and administer 50% dextrose without delay; he may die without it. If his BGL is normal (70 to 120 mg/dL), consider other conditions that may explain his presentation (eg, drug ingestion, poisoning, hypoxia). Further assess the patient, begin appropriate treatment, and safely move him to the ambulance. If he becomes violent, and hypoglycemia has been ruled out, give 2 to 5 mg of haloperidol (Haldol) IM. Your goal is to care for the patient while keeping you and your partner safe at the same time.

You respond to a lake at a state park where bystanders pulled a 44-year-old male from the water and are performing CPR on him. According to a park ranger, the water temperature of the lake is 85°F. No one witnessed the patient's submersion; however, one of his friends tells you that he is a diabetic and had been drinking beer all day. The cardiac monitor reveals a wide-complex bradycardic rhythm. In addition to CPR, further treatment should include: A: manual gastric decompression, insertion of an advanced airway device, vascular access, 1 mg of epinephrine, and 25 g of 50% dextrose. B: spinal precautions, insertion of an advanced airway device, hyperventilation, vascular access, 1 mg of epinephrine, and 50 mEq of sodium bicarbonate. C: cardiac pacing, intubation, gastric tube insertion, vascular access, 40 units of vasopressin, thermal management, and 25 g of 50% dextrose. D: spinal precautions, intubation, gastric tube insertion, vascular access, 1 mg of epinephrine, blood glucose assessment, and thermal management.

*D: spinal precautions, intubation, gastric tube insertion, vascular access, 1 mg of epinephrine, blood glucose assessment, and thermal management* Reason: Full spinal precautions are clearly indicated, especially following an unwitnessed submersion. When a swimmer panics, large amounts of water are swallowed; this causes gastric distention, which increases the risk of aspiration and makes ventilations difficult to perform. Intubate the patient and then insert a gastric tube to decompress the stomach. Following intubation, perform asynchronous CPR and deliver 8 to 10 breaths/min. Do not hyperventilate! Manual gastric decompression is dangerous and should be avoided, especially in nonintubated patients. Rescue airway devices (ie, King LT, LMA, CobraPLA) may provide better ventilation than a bag-mask device, but they do not eliminate the risk of aspiration. Establish vascular access and give 1 mg of epinephrine 1:10,000. A single dose of vasopressin (40 units) can be given to replace the first or second dose of epinephrine, but not both. There is currently no evidence to support the use of transcutaneous cardiac pacing (TCP) in cardiac arrest. Because the patient is a diabetic, assess his blood glucose level early and give 50% dextrose if he is hypoglycemic. Keep the patient warm; remove wet clothes and apply warm blankets. Treat suspected acidosis with adequate ventilation first, then contact medical control or follow your local protocols regarding the administration of sodium bicarbonate.

Increased parasympathetic tone, bradycardia, shunting of blood to the brain, and hypotension describes: A: Cushing's reflex. B: Beck's triad. C: Cullen's sign. D: the diving reflex.

*D: the diving reflex* Reason: The diving reflex, also known as the mammalian diving reflex, is a protective mechanism of the body that is most prominent in cold temperatures (ie, falling in cold water). Through increased parasympathetic tone, the pulse rate and blood pressure both fall to decrease overall oxygen demand and consumption, while at the same time, blood is shunted to the brain to sustain it for as long as possible. The diving reflex is the reason why small children are able to survive for extended periods of time when submerged in cold water. The effect of the diving reflex diminishes with age. Cushing's reflex, also called Cushing's triad, is a trio of clinical signs in patients with increased intracranial pressure; it includes hypertension, bradycardia, and abnormal breathing. Cullen's sign, bruising around the umbilicus, is an indicator of blood in the peritoneal space. Beck's triad, a trio of clinical signs observed in patients with a severe pericardial tamponade, includes muffled or distant heart tones, jugular venous distention, and a narrowing pulse pressure.

A 59-year-old woman with a history of Grave's disease presents with an altered mental status. Her skin is hot to the touch and her pulse rate is 160 beats/min. These findings are MOST consistent with: A: Cushing syndrome. B: myxedema. C: addisonian crisis. D: thyrotoxic crisis.

*D: thyrotoxic crisis* Reason: Thyrotoxic crisis (thyroid storm, thyrotoxicosis) is a hypermetabolic clinical syndrome caused by critically high levels of the thyroid hormones T3 (triiodothyronine) and T4 (thyroxine). Signs and symptoms include high fever (as high as 105°F to 106°F [40.5°C to 41.1°C]), hypertension, and profound tachycardia. Untreated, it can lead to cardiac arrest. Thyrotoxic crisis may occur in conjunction with Grave's disease, the most severe and common cause of hyperthyroidism, or it may occur if a patient takes too much of their prescribed thyroid supplement (ie, levothyroxine). Advanced hypothyroidism (myxedema) is a hypometabolic clinical syndrome caused by a deficiency of T3 and T4; the patient's signs and symptoms are not consistent with myxedema. Addisonian crisis, an acute manifestation of adrenal insufficiency, typically occurs after the abrupt cessation of corticosteroid therapy (ie, prednisone, hydrocortisone); this would not explain the patient's hypermetabolic state. Cushing syndrome, caused by excessive cortisol production by the adrenal glands, can also cause a hypermetabolic state. However, given the patient's history of Grave's disease, this is less likely.

Which of the following combinations of drugs are indicated for a patient with copious bronchial secretions, marked bradycardia, and profuse diaphoresis following exposure to an industrial pesticide? A: Pralidoxime chloride and diazepam B: Atropine and pralidoxime chloride C: Epinephrine and 2-PAM chloride D: Atropine and physostigmine

B: Atropine and pralidoxime chloride Reason: Organophosphate and carbamate pesticides are cholinergic agents that stimulate the parasympathetic nervous system (PNS) by deactivating acetylcholinesterase (AChE). AChE is an enzyme that regulates the degradation of acetylcholine (ACh), the chemical neurotransmitter of the PNS. Without AChE, ACh accumulates and causes profound stimulation of the PNS. The mnemonic DUMBELS is useful for remembering the signs and symptoms of organophosphate and carbamate poisoning; it stands for defecation, urination, miosis (pupillary constriction), bradycardia and bronchorrhea, emesis, lacrimation, and salivation. Atropine sulfate—an anticholinergic agent—is the first-line drug used in the treatment of organophosphate/carbamate poisoning. Atropine competitively antagonizes the action of ACh and reverses its muscarinic effects (ie, bradycardia, excessive secretions). Atropine is given in a dose of 1 to 2 mg every 5 to 15 minutes until secretions clear and the heart rate increases; higher doses are often required for patients with severe toxicity. Pralidoxime chloride (2-PAM chloride, Protopam) is given after atropine to reactivate AChE, allowing it to resume its normal function. The dose is 600 mg via autoinjector or 1 to 2 g IV over 15 to 30 minutes. Atropine and pralidoxime are commonly given together via autoinjector (Duodote). Seizures are common in patients with organophosphate/carbamate poisoning, and should be terminated with a benzodiazepine (ie, lorazepam [Ativan], diazepam [Valium]). Physostigmine is an AChE inhibitor that interferes with the metabolism of ACh. Because it inhibits the action of AChE, physostigmine is clearly not indicated and would only exacerbate the patient's condition.

You are called to a residence for a woman who swallowed an unknown quantity of pills. Which of the following should you inquire about FIRST? A: The patient's weight in kilograms B: What kind of pills were taken C: A history of psychiatric care D: When the pills were taken

B: Orotracheal intubation and 0.1 mg of epinephrine *1:10,000 IV or IO* Reason: The patient's airway is rapidly swelling, as evidenced by the stridorous respirations. Additionally, his level of consciousness and vital signs are consistent with shock. His airway is in immediate jeopardy and must be secured before it closes completely. You should insert an endotracheal tube (a smaller than normal tube may be needed) via the orotracheal route and assist his ventilations, establish IV or IO access, and administer 0.1 mg (1 mL) of epinephrine 1:10,000; an epinephrine infusion may be needed for refractory anaphylaxis. Epinephrine by the SC route will be much less effective in this patient because he is in shock and peripheral perfusion is likely minimal. If you cannot successfully intubate him via the orotracheal route, you will likely not be able to intubate him via the nasotracheal route. Therefore, if orotracheal intubation is unsuccessful, you should proceed with a cricothyrotomy at once.

Which of the following findings is MOST suggestive of myxedema? A: Tachycardia B: Weight loss C: Hyperactivity D: Hypothermia

D: Hypothermia* Reason: Advanced hypothyroidism is sometimes called myxedema. Frequently, patients have localized accumulations of mucinous material in the skin, which gives the disease its name (the prefix myx- refers to "mucin," and edema means "swelling"). Myxedema manifests as a general slowing of the body's metabolic processes due to a significant reduction or absence of the thyroid hormones T3 (triiodothyronine) and T4 (thyroxine). Since the thyroid gland regulates the metabolic rate and metabolism produces heat energy, patients with myxedema are prone to hypothermia. This also explains why patients with hypothyroidism are poorly tolerant of cold temperatures. Other signs and symptoms of myxedema include lethargy, depression, bradycardia, and weight gain. In severe cases, coma and death can occur.


Conjuntos de estudio relacionados

Georgia Real Estate - Section 16 Unit 1

View Set

MCAT- Chemical and Physical Foundations of Biological Systems

View Set

Senderos 1 NN En La Clase Part 3

View Set

Ch 13: Normal and Altered Immune Responses

View Set

Drivers Ed Segment 2: Study Guide

View Set